Tackling the AP English Language and Composition essays: part 1

Statistical Mediation & Moderation in Psychological Research-Apr-06-2021-08-55-30-55-PM

So, what are the three AP Lang Essays? The College Board shares a lot of general information about these essays on its website, as well as a large number of excellent sample essays. I suggest you take the time to review all of that material, here. But here’s my primer:

On the AP Lang Exam, there are three essays to write, all in a row (during the second half of the exam, after an initial multiple-choice portion). They are:

  • The Synthesis Essay: You’ll be given a general topic or question for debate (like: should public libraries continue to exist? Or: is eminent domain just?). Multiple short sources taking positions on that topic will follow the prompt. You will then be asked to write your own, short essay taking a position on the topic, citing at least three of the sources that you read.
  • The Rhetoric Essay: You’ll be given a short, rhetorically interesting passage, either taking a position on a topic, telling a story, or performing some other function. You will then be asked to write a short essay analyzing this passage’s use of language/rhetorical approach.
  • The Argument Essay: You will be given some position, usually stated in some brief excerpt from an author’s work. For example, you might be given an excerpt from Proust that suggests that people often regret their choices, or an excerpt from Eleanor Roosevelt praising the virtue of courage. You will then be asked to take your own position on the topic. This time, you won’t be given sources to help you make your arguments; all of your arguments must come from your own brain. 

The scoring rubric for each essay is roughly similar, with six possible points awarded: there is one point for argument, four points for evidence and analysis, and one point for “sophistication.” What this means is that, in brief, you need to do three things on every essay to get a perfect score: 

  • Have an argument.
  • Back up your argument with evidence and analyze how that evidence supports your argument.
  • Have an ineffable, excellent quality to your writing, a sort of dexterity of mind and language, for which the scorers have reserved one, sacred point. 

You can’t really control whether or not you can achieve #3, and a lot of that will be based on your prior level of experience writing/reading; but you can control whether or not you achieve #1-2. So, a high score is totally within your power! The TLDR version of this post is: make a clear argument and back it up with concrete, analyzed evidence. But, of course, that’s not as easy as it looks, and I have many more thoughts on how to actually achieve it, and achieve it well...

The six major components of successfully writing a timed essay on an exam are:

  • Organizing your time
  • Reading and Annotating
  • Outlining Part 1: Thesis
  • Outlining Part 2: Structure
  • Writing Part 1: Paragraphs (Intro, Evidence, Analysis, Conclusion)
  • Writing Part 2: Sentence by Sentence

#1 Organizing your time

On the AP Lang exam, you get a total of 2 hours and 15 minutes to write your three essays. This time is split into chunks. First, there is a 15 minute “reading period”; next, there is a 2 hour “writing period.” What this seems to imply is that the exam would like you to read all of the questions and their supplemental texts (the Synthesis Essay question and texts, the Rhetoric Essay question and passage, the Argument Essay question and short question blurb) in the 15 minute reading period, and then proceed to write the essays, in response, in the two hour writing period. This, however, is obviously an insane approach. For one thing, it’s kind of impossible: no one could keep the details of three different essay questions and associated readings together in their head all at once. For another, it’s really time inefficient: if you read all the material for all three essays first, you’re going to have to go back to it, a lot, each time you start to write a new essay, to jog your memory. Basically, no one in their right mind would (or does) advise this approach. And even the College Board seems to know it makes no sense, because they allow you to continue reading and referring to the questions and texts after the reading period. 

What you should do instead? Simply treat the whole 2 hours and 15 minutes as a single time block. Divide it into three units of 45 minutes. Then, read and answer each of the three questions one after the other, giving 45 minutes to each. Start with the Synthesis Essay, followed by the Rhetoric Essay, and then the Argument Essay. 

Your process should look like this: during the 15 minute reading period, begin work on the Synthesis Essay by reading the question and texts and planning that essay. Then, when the 2-hour timer starts, devote the first 30 minutes to actually writing that essay. Next, spend 45 minutes reading the Rhetoric Essay question and passage, and writing the Rhetoric Essay. Finally, spend the last 45 minutes reading the Argument Essay question/blurb and then writing the Argument Essay. The Argument Essay should actually take you less time than the first two, which means you should end up with 5-10 minutes to proofread your other essays. That said, I advise that you leave time at the end of each 45-minute block to check over each individual essay. 

Now let’s talk about the Rhetoric Essay in particular. How should you organize your 45 minutes here? I suggest mapping out your time roughly like this: take about ten minutes to read the passage, take notes, and brainstorm; then, take about five minutes to make an outline for your essay; next, take about twenty to twenty-five minutes to write. Leave an extra five to seven minutes at the end to re-read and edit your work. As you practice, you might notice that slightly different divisions of time work best for you – feel free to be flexible! You don’t have to stick to your timetable exactly . BUT you should try to stick to a version of this timetable so that you have enough time for each of the steps. How? Watch the clock!

#2 Reading and Annotating

The Rhetoric Essay asks you to analyze the language or rhetoric that a passage uses to achieve its ends. In your first ten minutes of reading, you should be keeping an eye out for two things: 

  • What is this passage trying to achieve? Is it trying to persuade the reader of an argument (often the case)? Is it trying to entertain the reader with a story (sometimes the case)? Is it trying to make the reader laugh? Is it trying to make the reader think? Identify the passage’s main purpose.
  • What rhetorical methods or devices does the passage use to achieve its aims? What exactly is it doing to achieve its aims? Yes, you should be watching out for rhetorical devices that already have fancy names, like “allusion” or “alliteration,” but you should also be using your OWN language/descriptive powers to identify the passage’s methods. You might, for example, note things like: “makes argument largely through anecdote” or “addresses counterarguments” or “lists so many absurd situations that they start to feel normal.” Try to identify not just rhetorical methods the passage uses, but also the central ones it uses.

To achieve this, I suggest proceeding as follows: read one paragraph. Once you’re done, stop, reflect, and note (in the margins) the most important rhetorical devices the passage used to achieve its aims (as far as you understand them thus far). Do this for each paragraph you read. Once you’re done, you should have a handy list in the margin of rhetorical tactics the passage uses. Which ones, looking back, seem to come up the most frequently? Which ones, even if they don’t come up frequently, seem particularly central to the passage’s aims? The tactics you identify will soon play a role in your essay’s thesis. 

Next, you’ll be ready to write an outline for your essay, mapping out (as best you can) its thesis and structure. In the next blog post , we’ll begin with that step. 

Related Content

AP ® Lang teachers: looking to help your students improve their rhetorical analysis essays?

Coach Hall Writes

clear, concise rhetorical analysis instruction.

AP Lang Exam FAQ

April 28, 2022 by Beth Hall

As an AP Lang teacher and someone who talks about AP® Lang on YouTube quite a bit, I have heard several frequently asked questions about the AP Lang test. In this post, I want to answer those AP Lang exam FAQ because chances are you might be wondering about some of them as well!

Don’t see your question here? Check out my YouTube channel or blog for way more AP®AP  Lang content.

Can I move around, or skip a question and come back?

You are able to move around between questions within a section. What that means is you won’t be able to see the essays while completing the multiple choice, but you can navigate between multiple choice passages and questions, or even between essays (when you are in that section).

Before you even read the passage, take a look at the questions (not the answer choices), so you know what to look for as you read. Additionally, you have limited time, don’t linger too long on any one particular question.

Do I get penalized for guessing on the AP Lang exam?

No, there is no penalty for guessing. If you are running out of time in the multiple choice section, choose one letter and bubble that answer consistently on the questions you won’t have time for. You should aim to answer every question, even if it is a guess.

How should I use my time during the AP Lang essays?

During the essay section, you will get 2 hours and 15 minutes to complete 3 essays. The first 15 minutes is a reading time, but if you are ready to begin planning or writing, then go for it.

Then, take a look at the 3 essays. If there is an essay topic you think is easy or an essay type you normally do well on, start with that. This allows you plenty of time to complete the essay as satisfactorily as possible. 

When thinking of time, you want to allot about 40 minutes per essay. The bulk of the time will be spent writing the body paragraphs. Don’t worry too much about spelling and grammar since these items generally do not affect your score since the essays are considered rough drafts. If you do edit something, a simple strikethrough will do.

For more information about planning how to spend your time on the AP Lang exam, check out this video about making your AP Lang game plan.

How many paragraphs do you need on the AP Lang essay?

You want to have an intro paragraph, but it can be short and sweet. It can simply be a defensible thesis with a few sentences of context before (if time permits).

The general rule is quality is better than quantity. Two really good body paragraphs is better than three mediocre ones. Conclusions are nice but not necessary. If you have the time and you feel it would benefit your essay, then do it. If not, then skip it.

For tips about writing a rhetorical analysis introduction, check out this blog post.

What materials do I need for the AP Lang Test?

On the day of the test, you should bring at least two pencils for the multiple choice section and two pens (blue or black ink) for the essays. You may also bring a snack, depending on the testing site rules. A watch may also be a good idea, but it cannot be a smart watch. If you don’t have a watch, don’t worry–there should be a clock in the testing room.

How long is the AP Lang test?

The first portion of the test is multiple choice. You have 1 hour to answer 45 questions. The second half is the essays. You have 2 hours and 15 minutes for 3 essays. This amounts to a 15-minute recommended reading period and 40 minutes for each of the essays.

How do you earn the sophistication point on the AP Lang exam?

I have a video on YouTube all about the sophistication point that you can watch here . I recommend looking at samples from The College Board and reading the ones that have earned a sophistication point. Look at the essay commentary to see why they earned the point.

Can I write about personal experience in my AP Lang essay?

For a rhetorical analysis essay, no. However it can be used as an argument essay. Be sure the personal experience is relevant to the prompt and your thesis. It should be a meaningful piece of evidence that further develops your argument. For more tips, check out the video below.

Want more of your AP Lang exam FAQ answered? You can find more information about the AP Lang exam by checking out my YouTube channel here or my blog here!

AP® Lang Teachers

Looking to help your students improve their rhetorical analysis essays?

Latest on Instagram

ap lang essays time

Shop My TPT Store

AP English Language and Composition

Learn all about the course and exam. Already enrolled? Join your class in My AP.

Not a Student?

Go to AP Central for resources for teachers, administrators, and coordinators.

About the Exam

The AP English Language and Composition Exam will test your understanding of the concepts covered in the course units, as well as your ability to analyze texts and develop written arguments.  

New for 2024-25: MCQs Will Have Four Answer Choices

Starting in the 2024-25 school year, AP English Language and Composition multiple-choice questions (MCQs) will have four answer choices instead of five. This change will take effect with the 2025 exam. All resources have been updated to reflect this change.   

Wed, May 14, 2025

AP English Language and Composition Exam

This is the regularly scheduled date for the AP English Language and Composition Exam.

Exam Components

Section 1: multiple choice.

45 questions 45% of Score

Excerpts from nonfiction texts are accompanied by several multiple-choice questions:

  • 23–25 Reading questions: You’ll be asked to read and analyze nonfiction texts.
  • 20–22 Writing questions: You’ll be asked to read like a writer and consider revisions to the text.

Section 2: Free Response

3 questions 55% of Score

The 2 hour and 15 minute time limit for this section includes a 15-minute reading period.

In the free-response section, you’ll respond to three questions with written answers. This section tests your skill in composition in three areas:

  • Synthesis: After reading 6 texts about a topic (including visual and quantitative sources), you will compose an argument that combines and cites at least 3 of the sources to support your thesis.
  • Rhetorical analysis: You will read a nonfiction text and analyze how the writer’s language choices contribute to the intended meaning and purpose of the text.
  • Argument: You will create an evidence-based argument that responds to a given topic.

Exam Essentials

Exam preparation, ap classroom resources.

Once you join your AP class section online, you’ll be able to access AP Daily videos, any assignments from your teacher, and your assignment results in AP Classroom. Sign in to access them.

  • Go to AP Classroom

Free-Response Questions and Scoring Information

Go to AP Central to review free-response questions and scoring information from past exams.

AP English Language and Composition Course and Exam Description

This is the core document for the course. It clearly lays out the course content and describes the exam and AP Program in general.

Services for Students with Disabilities

Students with documented disabilities may be eligible for accommodations for the through-course assessment and the end-of-course exam. If you’re using assistive technology and need help accessing the PDFs in this section in another format, contact Services for Students with Disabilities at 212-713-8333 or by email at [email protected] . For information about taking AP Exams, or other College Board assessments, with accommodations, visit the Services for Students with Disabilities website.

Credit and Placement

Search AP Credit Policies

Find colleges that grant credit and/or placement for AP Exam scores in this and other AP courses.

Additional Information

This site uses various technologies, as described in our Privacy Policy, for personalization, measuring website use/performance, and targeted advertising, which may include storing and sharing information about your site visit with third parties. By continuing to use this website you consent to our Privacy Policy and Terms of Use .

We are experiencing sporadically slow performance in our online tools, which you may notice when working in your dashboard. Our team is fully engaged and actively working to improve your online experience. If you are experiencing a connectivity issue, we recommend you try again in 10-15 minutes. We will update this space when the issue is resolved.

Enter your email to unlock an extra $25 off an SAT or ACT program!

By submitting my email address. i certify that i am 13 years of age or older, agree to recieve marketing email messages from the princeton review, and agree to terms of use., guide to the ap english language and composition exam.

AP English Language Exam

Can you apply the rhetorical triangle to a piece of writing? Are you able to argue a position? The AP ® English Language and Composition exam tests topics and skills discussed in your Advanced Placement English Language course. If you score high enough, your AP English Language score could earn you college credit!

Check out our AP English Language Guide for what you need to know about the exam:

  • Exam Overview
  • Sections and Question Types
  • How to Prepare

What’s on the AP English Language & Composition Exam?

The College Board is very detailed in what they require your AP teacher to cover in his or her AP English Language & Composition course. The exam tests your abilities to understand how authors use rhetoric and language to convey their purpose. Students are also expected to apply these techniques to their own writing and research projects. Some of the major skills tested include the ability to:

  • Identify an author’s purpose and intended audience
  • Recognize rhetorical devices and strategies in an author’s work
  • Demonstrate understanding of citations in research papers
  • Apply these skills and techniques to their own writing
  • Create and organize an argument defended with evidence and reasoning
  • Plan, write, and revise cogent, well-written essays

Check out our line of AP guides  for a comprehensive content review.

AP English Language Sections & Question Types

The AP English Language & Composition exam is 3 hours and 15 minutes long and consists of two sections: a multiple-choice section and a free response section.

Section 1

60 minutes

45 multiple choice questions

45%

Section 2

2 hours and 15 minutes

3 free response questions

55%

Read More: Review for the exam with our  AP English Language Crash Course 

Multiple-Choice

For AP English Language multiple-choice questions, you are presented with two Reading Passages and three Writing passages. The two Reading passages are nonfiction passages taken from all sorts of works. The idea is to get you to focus on rhetorical devices, figures of speech and intended purposes, under rigid time constraints and with material you haven’t seen before. The three Writing passages are student-produced essays. The idea is to get you to revise the essay that help the writer accomplish his or her goal.

Free Response

The AP English Language section contains three essay prompts: a synthesis essay, a rhetorical analysis essay, and an argument essay.

  • Synthesis essay: You’ll be given a scenario and tasked with writing a response using at least three of six or seven short accompanying sources for support.
  • Rhetorical analysis essay: Asks you to analyze the techniques an author uses, and discuss how they contribute to the author’s purpose.
  • Argument essay: Presents a claim or assertion in the prompt and then asks you to argue a position based on your own knowledge, experience, or reading.

How to Interpret AP English Language Scores

AP scores are reported from 1 to 5. Colleges are generally looking for a 4 or 5 on the AP English Language exam, but some may grant AP credit for a 3. Each test is curved so scores vary from year to year. Here’s how AP English Lang students scored on the May 2022 test:

5

Extremely qualified

10.4%

4

Well qualified

21.1%

3

Qualified

24.2%

2

Possibly qualified

29.8%

1

No recommendation

14.5%

Source: College Board

How can I prepare?

AP classes are great, but for many students they’re not enough! For a thorough review of AP English Language content and strategy, pick the AP prep option that works best for your goals and learning style.

  • AP Exams  

Explore Colleges For You

Explore Colleges For You

Connect with our featured colleges to find schools that both match your interests and are looking for students like you.

Career Quiz

Career Quiz

Take our short quiz to learn which is the right career for you.

Connect With College Coaches

Get Started on Athletic Scholarships & Recruiting!

Join athletes who were discovered, recruited & often received scholarships after connecting with NCSA's 42,000 strong network of coaches.

Best 389 Colleges

Best 389 Colleges

165,000 students rate everything from their professors to their campus social scene.

SAT Prep Courses

1400+ course, act prep courses, free sat practice test & events,  1-800-2review, free digital sat prep try our self-paced plus program - for free, get a 14 day trial.

ap lang essays time

Free MCAT Practice Test

I already know my score.

ap lang essays time

MCAT Self-Paced 14-Day Free Trial

ap lang essays time

Enrollment Advisor

1-800-2REVIEW (800-273-8439) ext. 1

1-877-LEARN-30

Mon-Fri 9AM-10PM ET

Sat-Sun 9AM-8PM ET

Student Support

1-800-2REVIEW (800-273-8439) ext. 2

Mon-Fri 9AM-9PM ET

Sat-Sun 8:30AM-5PM ET

Partnerships

  • Teach or Tutor for Us

College Readiness

International

Advertising

Affiliate/Other

  • Enrollment Terms & Conditions
  • Accessibility
  • Cigna Medical Transparency in Coverage

Register Book

Local Offices: Mon-Fri 9AM-6PM

  • SAT Subject Tests

Academic Subjects

  • Social Studies

Find the Right College

  • College Rankings
  • College Advice
  • Applying to College
  • Financial Aid

School & District Partnerships

  • Professional Development
  • Advice Articles
  • Private Tutoring
  • Mobile Apps
  • International Offices
  • Work for Us
  • Affiliate Program
  • Partner with Us
  • Advertise with Us
  • International Partnerships
  • Our Guarantees
  • Accessibility – Canada

Privacy Policy | CA Privacy Notice | Do Not Sell or Share My Personal Information | Your Opt-Out Rights | Terms of Use | Site Map

©2024 TPR Education IP Holdings, LLC. All Rights Reserved. The Princeton Review is not affiliated with Princeton University

TPR Education, LLC (doing business as “The Princeton Review”) is controlled by Primavera Holdings Limited, a firm owned by Chinese nationals with a principal place of business in Hong Kong, China.

ap lang essays time

Expert Guide to the AP Language and Composition Exam

ap lang essays time

The AP Language and Composition Exam is a comprehensive assessment of students' reading, writing, and critical thinking skills. Here is an expert guide to help you navigate and excel in this exam:

1. Exam Format: The AP Language and Composition Exam consists of multiple-choice questions and free-response tasks. The multiple-choice section tests your reading comprehension and analysis skills, while the free-response section assesses your ability to write coherent and persuasive essays.

2. Analyzing Rhetorical Strategies: A key focus of the exam is analyzing and understanding rhetorical strategies used in various texts. This includes identifying and evaluating techniques such as ethos, pathos, logos, and rhetorical devices like imagery, figurative language, and tone. Practice analyzing different types of texts, including speeches, articles, essays, and advertisements.

3. Essay Writing Skills: The free-response section requires you to write three essays: a synthesis essay, a rhetorical analysis essay, and an argument essay. Develop strong essay writing skills, including thesis development, evidence selection, and paragraph organization. Practice constructing well-structured, coherent, and persuasive arguments within the given time constraints.

4. Close Reading and Annotation: Effective close reading and annotation skills are crucial for success in the exam. Learn to identify the main ideas, key details, and rhetorical elements in the provided passages. Annotate the text to mark important points, make connections, and track your understanding of the author's purpose and argument.

5. Vocabulary and Grammar: Enhance your vocabulary and grammar skills to express your ideas clearly and precisely. Use varied and appropriate language to convey your analysis and arguments effectively. Pay attention to sentence structure, punctuation, and word choice to ensure coherence and precision in your writing.

6. Practice and Timed Mock Exams: Regular practice is essential to build your skills and confidence. Take timed mock exams to simulate the exam conditions and develop your time management skills. Review your performance, identify areas for improvement, and seek feedback from teachers or peers.

7. Read Widely: Expand your reading repertoire by engaging with diverse texts from different genres and time periods. Reading extensively will improve your comprehension, vocabulary, and ability to recognize different writing styles and rhetorical strategies.

8. Critical Thinking and Analysis: Develop your critical thinking skills by analyzing the effectiveness of arguments, evaluating evidence, and recognizing biases and logical fallacies. Practice constructing well-reasoned arguments and counterarguments to strengthen your analysis.

9. Stay Updated with Current Events: Stay informed about current events and societal issues as they often form the basis of essay prompts and analysis passages. Familiarize yourself with contemporary debates, social, and political issues, and be prepared to apply your knowledge to the exam questions.

10. Seek Resources and Guidance: Utilize available resources, such as study guides, practice exams, and online resources, to enhance your preparation. Seek guidance from teachers, tutors, or peers to clarify any doubts and improve your understanding of the exam requirements.

The AP English Language and Composition Multiple-Choice

The multiple-choice section of the AP English Language and Composition exam assesses your reading comprehension and analysis skills. Here are some key points to understand and excel in this section:

1. Format and Structure: The multiple-choice section consists of a series of passages followed by a set of questions. The passages can include a variety of genres such as essays, speeches, articles, and excerpts from books or plays. Each passage is accompanied by multiple-choice questions that require you to analyze the author's purpose, rhetoric, and style.

2. Close Reading: Effective close reading is crucial for success in the multiple-choice section. Read the passages carefully, paying attention to details, tone, and the author's use of rhetorical devices. Underline or annotate important sections to help you remember key points and refer back to them when answering the questions.

3. Understanding Rhetorical Devices: Familiarize yourself with common rhetorical devices such as ethos, pathos, logos, irony, figurative language, and tone. These devices are frequently used by authors to convey their message and persuade the reader. Be prepared to identify and analyze how these devices contribute to the author's overall argument or purpose.

4. Analyzing Text Structure: Pay attention to the structure of the passages, including the organization of ideas, transitions, and the use of evidence. Identify the main idea, supporting details, and the logical flow of the author's argument. Understanding the structure of the passage will help you answer questions related to the author's intent and the development of their ideas.

5. Answering Strategies: Develop effective strategies for approaching multiple-choice questions. Read each question carefully, making sure to consider all the answer choices before selecting the best option. Pay attention to qualifiers such as "most likely," "least likely," "best supports," etc. Eliminate clearly incorrect choices and make an educated guess if you are unsure.

6. Time Management: The multiple-choice section is timed, so it is important to manage your time effectively. Pace yourself and allocate a specific amount of time for each passage and its corresponding questions. If you encounter a challenging question, mark it and move on, returning to it later if time permits.

7. Practice with Sample Questions: Familiarize yourself with the types of questions commonly asked in the AP English Language and Composition exam by practicing with sample questions. This will help you become more comfortable with the format and style of the questions and improve your ability to identify key elements in the passages.

8. Review Test-Taking Strategies: In addition to content knowledge, review general test-taking strategies that can improve your performance. This includes strategies for eliminating answer choices, using process of elimination, and managing your time effectively.

The AP English Language and Composition Free Response

The free response section of the AP English Language and Composition exam is designed to assess your ability to analyze and respond to rhetorical prompts effectively. Here are some key points to understand and excel in this section:

1. Format and Structure:

The free response section consists of three essay prompts: a synthesis essay, a rhetorical analysis essay, and an argument essay. Each prompt presents you with a specific task and requires you to analyze and respond to a given passage or passages.

2. Synthesis Essay:

In this essay, you are asked to combine information from multiple sources to create a coherent and well-supported argument. You must demonstrate your ability to understand and synthesize different perspectives on a given topic. It is important to analyze the sources critically, identify their main arguments, and use evidence from the sources to support your own argument.

3. Rhetorical Analysis Essay:

In this essay, you are required to analyze the rhetorical strategies employed by the author of a given passage. You need to identify and explain the author's use of rhetorical devices, such as ethos, pathos, logos, figurative language, and tone. Your analysis should focus on how these devices contribute to the author's overall argument and purpose.

4. Argument Essay:

In this essay, you are expected to construct and support your own argument on a given topic. You must develop a clear and coherent thesis statement, provide relevant evidence, and effectively address counterarguments. It is important to use persuasive techniques and rhetorical devices to strengthen your argument.

5. Organization and Structure:

Structure your essays in a clear and logical manner. Each essay should have an introduction that presents your thesis statement, body paragraphs that support your thesis with evidence and analysis, and a conclusion that summarizes your main points and reinforces your argument. Use topic sentences and transitions to ensure a smooth flow of ideas.

6. Evidence and Analysis:

Support your claims and arguments with evidence from the given passages or external sources. Use specific examples, quotes, and references to demonstrate your understanding and provide strong evidence for your analysis. Avoid making unsupported generalizations or relying solely on personal opinions.

7. Time Management:

The free response section is time-limited, so it is crucial to manage your time effectively. Allocate a specific amount of time for each essay and stick to it. Leave some time at the end to review and revise your essays for clarity, coherence, and grammatical correctness.

8. Practice and Preparation:

Familiarize yourself with the expectations and requirements of each essay type by practicing with past exam prompts and sample essays. Pay attention to the scoring guidelines provided by the College Board to understand how your essays will be evaluated. Seek feedback from teachers or peers to improve your writing skills and address any weaknesses.

AP English Language Prep Tips

Preparing for the AP English Language exam requires a strategic approach to enhance your reading, writing, and analytical skills. Here are some detailed tips to help you excel in your preparation:

1. Read Widely:

Develop a habit of reading a variety of texts, including fiction, non-fiction, essays, newspaper articles, and editorials. This will expose you to different writing styles, perspectives, and rhetorical devices. Pay attention to the author's tone, purpose, and argumentative strategies.

2. Analyze Rhetorical Devices:

Familiarize yourself with common rhetorical devices such as ethos, pathos, logos, figurative language, and rhetorical appeals. Practice identifying these devices in various texts and analyze how they contribute to the author's message and overall effectiveness.

3. Expand Vocabulary:

Enhance your vocabulary by reading challenging texts and keeping a vocabulary notebook. Learn new words, their definitions, and how they are used in context. Utilize these words in your writing to demonstrate a strong command of language.

4. Practice Timed Writing:

Time yourself while writing essays to simulate the exam conditions. Aim to complete essays within the time limit while maintaining clarity and coherence. Practice different essay types, such as synthesis, rhetorical analysis, and argument essays, to strengthen your skills in each area.

5. Read Sample Essays:

Study well-written sample essays from previous AP exams to understand the expectations and scoring criteria. Analyze their structure, use of evidence, and clarity of argument. Take note of effective introductions, strong thesis statements, and well-supported analysis.

6. Develop Writing Strategies:

Learn to effectively structure your essays with clear introductions, body paragraphs, and conclusions. Use topic sentences, transitions, and evidence to support your claims. Craft strong thesis statements that clearly state your position and guide your essay.

7. Analyze Visual Texts:

Practice analyzing visual texts such as graphs, charts, and images. Understand how visual elements convey information, make arguments, and support claims. Pay attention to the intended audience and the overall impact of visual texts.

8. Practice Multiple-Choice Questions:

Regularly practice multiple-choice questions to improve your reading comprehension and analysis skills. Read passages carefully, annotate as you go, and answer questions based on the given information. Pay attention to details, context, and authorial intent.

9. Seek Feedback:

Share your essays with teachers or peers and seek constructive feedback. Learn from their suggestions to improve your writing skills and address any weaknesses. Consider joining or forming study groups to discuss and analyze different texts and essay prompts.

10. Review Grammar and Mechanics:

Brush up on grammar rules and punctuation to ensure your writing is clear and error-free. Pay attention to sentence structure, verb tense, subject-verb agreement, and pronoun usage. A strong command of grammar enhances the clarity and effectiveness of your writing.

Remember that consistent practice, focused study, and critical reading are key to success in the AP English Language exam. Develop a study schedule, allocate time for reading and writing practice, and stay disciplined in your preparation. With dedication and effort, you can improve your skills and perform well on the exam.

AP Language and Composition Test Day Tips

On the day of the AP Language and Composition exam, it's important to be well-prepared and approach the test with confidence. Here are some detailed tips to help you make the most of your test day:

1. Get a Good Night's Sleep:

Ensure you have a restful night's sleep before the exam day. Being well-rested will help you stay focused and maintain mental clarity throughout the test.

2. Eat a Nutritious Breakfast:

Start your day with a healthy and balanced breakfast. Fueling your body with nutritious food will provide you with the energy you need for the duration of the exam.

3. Arrive Early:

Plan to arrive at the exam location early to avoid any unnecessary stress. Familiarize yourself with the exam venue and locate your assigned room beforehand.

4. Bring Necessary Materials:

Double-check that you have all the required materials for the exam, such as your admission ticket, identification, pens, pencils, erasers, and a watch to keep track of time. Be aware of any specific items allowed or prohibited by the testing guidelines.

5. Read Instructions Carefully:

Take the time to carefully read the instructions provided on the exam booklet and answer sheet. Understand the format, timing, and specific requirements for each section of the test.

6. Pace Yourself:

Time management is crucial in the AP Language and Composition exam. Allocate your time wisely, making sure to complete each section within the specified time limits. Pace yourself and avoid spending too much time on any single question or passage

7. Skim the Questions First:

Before diving into the reading passages, quickly skim the multiple-choice questions to get a sense of what to look for as you read. This can help you focus your attention and save time while reading and analyzing the passages.

8. Read Actively and Annotate:

As you read the passages, actively engage with the text. Underline key points, annotate important details, and mark passages that you may want to refer back to later. This will help you remember crucial information and facilitate your analysis.

9. Plan Your Essays:

For the essay sections, take a few minutes to plan your response before writing. Outline your main points, supporting evidence, and a clear thesis statement. This will provide structure to your essay and ensure a more coherent and organized response.

10. Review Your Work:

If time permits, take a moment to review your answers before submitting your exam. Check for any errors or incomplete responses, and make any necessary corrections or additions. Ensure that you have followed the instructions and provided clear and concise answers.

11. Stay Calm and Focused:

Throughout the exam, maintain a calm and focused mindset. Manage test anxiety by taking deep breaths, maintaining a positive attitude, and focusing on the task at hand. Remember that you have prepared for this exam and trust in your abilities.

12. Follow Exam Regulations:

Adhere to the exam regulations and guidelines provided by the College Board. Maintain academic integrity by refraining from any prohibited behavior, such as cheating or using unauthorized materials.

By following these tips, you can approach the AP Language and Composition exam with confidence and maximize your chances of success. Remember to stay calm, trust your preparation, and showcase your skills in analyzing and responding to complex texts. Good luck!

In conclusion, the AP Language and Composition exam can seem challenging, but with the right preparation and approach, you can excel. Understanding the exam format, practicing multiple-choice questions, mastering the free response section, and developing strong analytical and writing skills are essential for success. Additionally, following test day tips and maintaining a calm and focused mindset will help you perform at your best. By leveraging these insights and strategies, you can navigate the AP Language and Composition exam with confidence and achieve a high score. Good luck on your exam!

You Might Also Like

ap lang essays time

How to Write a Recommendation Letter for College Admissions

Learn some tips that you can do to ensure that your recommendation letter get accepted and you can get admission in your dream university/college - Read a blog

ap lang essays time

The Perfect College Essay Structure

The Fundamentals of writing an Essay which includes the process of brainstorming, drafting, and finalizing.

ap lang essays time

The Ultimate Guide to College Interviews

College interviews, although nerve wrecking, can be the best chance for you to impress the admissions officer. We give you tips on how to ace your interview.

AP Guru has been helping students since 2010 gain admissions to their dream universities by helping them in their college admissions and SAT and ACT Prep

Free Resources

logo-type-white

AP® English Language

How long is the ap® english language exam tips to manage your time for a 5.

  • The Albert Team
  • Last Updated On: April 22, 2022

how_long_is_the_ap_english language exam

The AP® English Language exam tests your knowledge of English language and focuses on rhetorical analysis of nonfiction texts, along with the development and revision  of well-reasoned, evidence-centered analytic and argumentative writing.

To help you prepare, let’s break down the AP® English Language exam into sections and look at some tips on how to tackle each section effectively under the given time constraints — AP® test time management is crucial to success on the exam.

How Long is the AP® English Language Exam?

The AP® English Language exam has two sections. The total exam is three hours and 15 minutes long. The details of each section and part are outlined in the table below:

52 – 55 Questions

1 hour

45% of total exam score

3 Questions

2 hours, 15 minutes minutes (including a 15-minute reading period)

55% of total exam score

First, let’s look at Section I in greater detail to help you prepare for this first part of the exam.

How Long is the AP® English Language Exam Section I?

The first part of Section I of the AP® English Language Exam is Multiple Choice. You will have one hour to answer 52-55 questions, which works out to slightly over one minute per question. This part accounts for 45% of your total exam score.

Here, you are given excerpts from various non-fiction texts, each of which are accompanied by several multiple-choice questions. In the past, students have received four or five passages, each with approximate 10 questions to answer.

The questions can be broken down into eight buckets:

  • Reading Comprehension
  • Implication
  • Overall Passage Analysis
  • Relationship Between Parts of Text
  • Interpretation of Figurative Language
  • Purpose of Part of Text
  • Rhetorical Strategy
  • Style and Effect

It is in your best interest to practice all of theses to determine which areas your are weaker in.

How to Manage Your Time in Section I of the AP® English Language Exam

While the exam structure may appear daunting, there are some things you should keep in mind to maximize your score on Section I:

  • It is important to recognize that the multiple-choice questions associated with each reading are independent. If you are having trouble answering some questions associated with one reading, don’t waste your valuable time struggling. Remember, you don’t have much more than one minute per question! Instead, move on to the next passage, where you may have more success in answering questions correctly. Cut your losses when you have to, and keep in mind that you can always double back to work on skipped questions if you have enough time. And in case you can’t figure out the correct answer, just take an educated guess, as there is no penalty for incorrect answers.
  • Read the questions briefly before diving into the passages. Often, the passages can be dense and contain information that is not necessary to answer the subsequent multiple-choice questions. By reading the questions and the associated answer choices, you will have a sense of what themes to search for when you are reading. If there are questions asking for analysis of a certain line in the passage, you will know in advance to read that line closely. As a result, you will hopefully be able to more easily isolate the answers to certain questions without having to backtrack and comb through the passage again. This should help you avoid spending too much time on one question.

Hopefully, these tips will help you with your AP® English Language test time management. Now, let’s break down Section II of the AP® English Language Exam.

How Long is the AP® English Language Exam Section II?

Section II of the AP® English Language Exam is the Free Response Section. As you would expect, you are not given any choices to select. You have 15 minutes to peruse the sources provided, and two hours to answer three questions using the evidence. You are allowed to begin writing during the reading period if you would like. This section accounts for 55% of your total exam score.

These questions will test your ability to quickly formulate arguments, form inferences, and craft analysis drawn from the sources provided to you. If you don’t understand early on how to go about following the instructions on the exam, you might find this portion more difficult than the multiple-choice section.

According to CollegeBoard, this section has three prompts:

  • Synthesis : Students read several texts about a topic and create an argument that synthesizes at least three of the sources to support their thesis.
  • Rhetorical analysis : Students read a nonfiction text and analyze how the writer’s language choices contribute to his or her purpose and intended meaning for the text.
  • Argument : Students create an evidence-based argument that responds to a given topic.

How to Manage Your Time in Section II of the AP® English Language Exam

time management

Free Response questions can be a little scary because you can’t guess if you’re not sure of yourself. However, we believe these AP® test time management tips will help you ace Section II:

  • Read the question multiple times to understand what is truly being asked. Getting to the core of the prompt will help you craft a concise thesis that serves as the centerpiece to your entire response.
  • Following the thesis, construct a road map that serves as a guide to your reader. It will serve as an outline for your subsequent paragraphs and conveys how they relate to your thesis. Organize the rest of your essay with topic sentences that directly follow from your thesis and provide a summary of the rest of the paragraph. Then, provide context, cite your evidence, and lastly, dive into the analysis that relates your evidence to your thesis. Following this strategy will develop a clear structure that will add clarity to your essay.
  • Rely on your sources to bolster your argument. They are provided to you so you use them — don’t neglect to reference all your sources because you may initially understand how they can be used. Take some time to isolate the main theme of each source and select a few quotes that complement your argument. Make sure that you avoid repeating yourself — instead of summarizing the evidence you provide, try to dig one level deeper to explain why you incorporated it into your essay and how it relates to your thesis.
  • Become familiar with the grading rubrics for the essay questions. With the 2020 redesign came new rubrics for the AP® Lang essay section. Previously, essays were scored using holistic rubrics, on a scale of 0-9. Starting with the 2019 exam, students’ essays will be graded with new analytic rubrics. Each essay is worth up to 6 points. Think of the new rubrics as a How To Guide to getting a 6 on each essay.

How to Practice Time Management for AP® English Language?

In addition to the AP® test time management tips provided above, you may find it helpful to consult this post, which outlines the ultimate list of tips for the AP® English Language exam. The post discusses the specific topics which will be covered on the exam, common types of questions and general strategies to help you solve them, along with a list of tips from AP® English Language teachers. And if you want to get some practice, check out these practice questions. The more you practice, the more familiarity you will build with different types of questions. Eventually, you will be able to identify which areas you are weaker in and can direct the bulk of your studying efforts to improving your understanding of those concepts.

Don’t forget to time yourself while you work on practice questions so you can test yourself on managing your time as well as reviewing concepts. Some multiple-choice questions are notorious for being significant time drains, which can cost you when answering the rest of Section I questions. If you find yourself stuck on a multiple-choice question for more than a couple of minutes, it may be in your best interest to cut your losses and utilize the process of elimination to guess the most likely answer. Don’t let one question you can’t solve prevent you from answering multiple questions you can.

Hopefully, these tips help with your AP® test time management. Best of luck with your exam!

Looking for AP® English Language practice?

Kickstart your AP® English Language prep with Albert. Start your AP® exam prep today .

Interested in a school license?​

Popular posts.

AP® Physics I score calculator

AP® Score Calculators

Simulate how different MCQ and FRQ scores translate into AP® scores

ap lang essays time

AP® Review Guides

The ultimate review guides for AP® subjects to help you plan and structure your prep.

ap lang essays time

Core Subject Review Guides

Review the most important topics in Physics and Algebra 1 .

ap lang essays time

SAT® Score Calculator

See how scores on each section impacts your overall SAT® score

ap lang essays time

ACT® Score Calculator

See how scores on each section impacts your overall ACT® score

ap lang essays time

Grammar Review Hub

Comprehensive review of grammar skills

ap lang essays time

AP® Posters

Download updated posters summarizing the main topics and structure for each AP® exam.

PrepScholar

Choose Your Test

  • Search Blogs By Category
  • College Admissions
  • AP and IB Exams
  • GPA and Coursework

How to Write a Perfect Synthesis Essay for the AP Language Exam

author image

Advanced Placement (AP)

body-pencil-sharpen-notebook-1

If you're planning to take the AP Language (or AP Lang) exam , you might already know that 55% of your overall exam score will be based on three essays. The first of the three essays you'll have to write on the AP Language exam is called the "synthesis essay." If you want to earn full points on this portion of the AP Lang Exam, you need to know what a synthesis essay is and what skills are assessed by the AP Lang synthesis essay.

In this article, we'll explain the different aspects of the AP Lang synthesis essay, including what skills you need to demonstrate in your synthesis essay response in order to achieve a good score. We'll also give you a full breakdown of a real AP Lang Synthesis Essay prompt, provide an analysis of an AP Lang synthesis essay example, and give you four tips for how to write a synthesis essay.

Let's get started by taking a closer look at how the AP Lang synthesis essay works!

Synthesis Essay AP Lang: What It Is and How It Works

The AP Lang synthesis essay is the first of three essays included in the Free Response section of the AP Lang exam.

The AP Lang synthesis essay portion of the Free Response section lasts for one hour total . This hour consists of a recommended 15 minute reading period and a 40 minute writing period. Keep in mind that these time allotments are merely recommendations, and that exam takers can parse out the allotted 60 minutes to complete the synthesis essay however they choose.

Now, here's what the structure of the AP Lang synthesis essay looks like. The exam presents six to seven sources that are organized around a specific topic (like alternative energy or eminent domain, which are both past synthesis exam topics).

Of these six to seven sources, at least two are visual , including at least one quantitative source (like a graph or pie chart, for example). The remaining four to five sources are print text-based, and each one contains approximately 500 words.

In addition to six to seven sources, the AP Lang exam provides a written prompt that consists of three paragraphs. The prompt will briefly explain the essay topic, then present a claim that students will respond to in an essay that synthesizes material from at least three of the sources provided.

Here's an example prompt provided by the College Board:

Directions : The following prompt is based on the accompanying six sources.

This question requires you to integrate a variety of sources into a coherent, well-written essay. Refer to the sources to support your position; avoid mere paraphrase or summary. Your argument should be central; the sources should support this argument .

Remember to attribute both direct and indirect citations.

Introduction

Television has been influential in United States presidential elections since the 1960's. But just what is this influence, and how has it affected who is elected? Has it made elections fairer and more accessible, or has it moved candidates from pursuing issues to pursuing image?

Read the following sources (including any introductory information) carefully. Then, in an essay that synthesizes at least three of the sources for support, take a position that defends, challenges, or qualifies the claim that television has had a positive impact on presidential elections.

Refer to the sources as Source A, Source B, etc.; titles are included for your convenience.

Source A (Campbell) Source B (Hart and Triece) Source C (Menand) Source D (Chart) Source E (Ranney) Source F (Koppel)

Like we mentioned earlier, this prompt gives you a topic — which it briefly explains — then asks you to take a position. In this case, you'll have to choose a stance on whether television has positively or negatively affected U.S. elections. You're also given six sources to evaluate and use in your response. Now that you have everything you need, now your job is to write an amazing synthesis essay.

But what does "synthesize" mean, exactly? According to the CollegeBoard, when an essay prompt asks you to synthesize, it means that you should "combine different perspectives from sources to form a support of a coherent position" in writing. In other words, a synthesis essay asks you to state your claim on a topic, then highlight the relationships between several sources that support your claim on that topic. Additionally, you'll need to cite specific evidence from your sources to prove your point.

The synthesis essay counts for six of the total points on the AP Lang exam . Students can receive 0-1 points for writing a thesis statement in the essay, 0-4 based on incorporation of evidence and commentary, and 0-1 points based on sophistication of thought and demonstrated complex understanding of the topic.

You'll be evaluated based on how effectively you do the following in your AP Lang synthesis essay:

Write a thesis that responds to the exam prompt with a defensible position

Provide specific evidence that to support all claims in your line of reasoning from at least three of the sources provided, and clearly and consistently explain how the evidence you include supports your line of reasoning

Demonstrate sophistication of thought by either crafting a thoughtful argument, situating the argument in a broader context, explaining the limitations of an argument

Make rhetorical choices that strengthen your argument and/or employ a vivid and persuasive style throughout your essay.

If your synthesis essay meets the criteria above, then there's a good chance you'll score well on this portion of the AP Lang exam!

If you're looking for even more information on scoring, the College Board has posted the AP Lang Free Response grading rubric on its website. ( You can find it here. ) We recommend taking a close look at it since it includes additional details about the synthesis essay scoring.

body-chisel-break-apart

Don't be intimidated...we're going to teach you how to break down even the hardest AP synthesis essay prompt.

Full Breakdown of a Real AP Lang Synthesis Essay Prompt

In this section, we'll teach you how to analyze and respond to a synthesis essay prompt in five easy steps, including suggested time frames for each step of the process.

Step 1: Analyze the Prompt

The very first thing to do when the clock starts running is read and analyze the prompt. To demonstrate how to do this, we'll look at the sample AP Lang synthesis essay prompt below. This prompt comes straight from the 2018 AP Lang exam:

Eminent domain is the power governments have to acquire property from private owners for public use. The rationale behind eminent domain is that governments have greater legal authority over lands within their dominion than do private owners. Eminent domain has been instituted in one way or another throughout the world for hundreds of years.

Carefully read the following six sources, including the introductory information for each source. Then synthesize material from at least three of the sources and incorporate it into a coherent, well-developed essay that defends, challenges, or qualifies the notion that eminent domain is productive and beneficial.

Your argument should be the focus of your essay. Use the sources to develop your argument and explain the reasoning for it. Avoid merely summarizing the sources. Indicate clearly which sources you are drawing from, whether through direct quotation, paraphrase, or summary. You may cite the sources as Source A, Source B, etc., or by using the descriptions in parentheses.

On first read, you might be nervous about how to answer this prompt...especially if you don't know what eminent domain is! But if you break the prompt down into chunks, you'll be able to figure out what the prompt is asking you to do in no time flat.

To get a full understanding of what this prompt wants you to do, you need to identify the most important details in this prompt, paragraph by paragraph. Here's what each paragraph is asking you to do:

  • Paragraph 1: The prompt presents and briefly explains the topic that you'll be writing your synthesis essay about. That topic is the concept of eminent domain.
  • Paragraph 2: The prompt presents a specific claim about the concept of eminent domain in this paragraph: Eminent domain is productive and beneficial. This paragraph instructs you to decide whether you want to defend, challenge, or qualify that claim in your synthesis essay , and use material from at least three of the sources provided in order to do so.
  • Paragraph 3: In the last paragraph of the prompt, the exam gives you clear instructions about how to approach writing your synthesis essay . First, make your argument the focus of the essay. Second, use material from at least three of the sources to develop and explain your argument. Third, provide commentary on the material you include, and provide proper citations when you incorporate quotations, paraphrases, or summaries from the sources provided.

So basically, you'll have to agree with, disagree with, or qualify the claim stated in the prompt, then use at least three sources substantiate your answer. Since you probably don't know much about eminent domain, you'll probably decide on your position after you read the provided sources.

To make good use of your time on the exam, you should spend around 2 minutes reading the prompt and making note of what it's asking you to do. That will leave you plenty of time to read the sources provided, which is the next step to writing a synthesis essay.

Step 2: Read the Sources Carefully

After you closely read the prompt and make note of the most important details, you need to read all of the sources provided. It's tempting to skip one or two sources to save time--but we recommend you don't do this. That's because you'll need a thorough understanding of the topic before you can accurately address the prompt!

For the sample exam prompt included above, there are six sources provided. We're not going to include all of the sources in this article, but you can view the six sources from this question on the 2018 AP Lang exam here . The sources include five print-text sources and one visual source, which is a cartoon.

As you read the sources, it's important to read quickly and carefully. Don't rush! Keep your pencil in hand to quickly mark important passages that you might want to use as evidence in your synthesis. While you're reading the sources and marking passages, you want to think about how the information you're reading influences your stance on the issue (in this case, eminent domain).

When you finish reading, take a few seconds to summarize, in a phrase or sentence, whether the source defends, challenges, or qualifies whether eminent domain is beneficial (which is the claim in the prompt) . Though it might not feel like you have time for this, it's important to give yourself these notes about each source so you know how you can use each one as evidence in your essay.

Here's what we mean: say you want to challenge the idea that eminent domain is useful. If you've jotted down notes about each source and what it's saying, it will be easier for you to pull the relevant information into your outline and your essay.

So how much time should you spend reading the provided sources? The AP Lang exam recommends taking 15 minutes to read the sources . If you spend around two of those minutes reading and breaking down the essay prompt, it makes sense to spend the remaining 13 minutes reading and annotating the sources.

If you finish reading and annotating early, you can always move on to drafting your synthesis essay. But make sure you're taking your time and reading carefully! It's better to use a little extra time reading and understanding the sources now so that you don't have to go back and re-read the sources later.

body-weightlifting-lift-strong

A strong thesis will do a lot of heavy lifting in your essay. (See what we did there?)

Step 3: Write a Strong Thesis Statement

After you've analyzed the prompt and thoroughly read the sources, the next thing you need to do in order to write a good synthesis essay is write a strong thesis statement .

The great news about writing a thesis statement for this synthesis essay is that you have all the tools you need to do it at your fingertips. All you have to do in order to write your thesis statement is decide what your stance is in relationship to the topic provided.

In the example prompt provided earlier, you're essentially given three choices for how to frame your thesis statement: you can either defend, challenge, or qualify a claim that's been provided by the prompt, that eminent domain is productive and beneficial . Here's what that means for each option:

If you choose to defend the claim, your job will be to prove that the claim is correct . In this case, you'll have to show that eminent domain is a good thing.

If you choose to challenge the claim, you'll argue that the claim is incorrect. In other words, you'll argue that eminent domain isn't productive or beneficial.

If you choose to qualify, that means you'll agree with part of the claim, but disagree with another part of the claim. For instance, you may argue that eminent domain can be a productive tool for governments, but it's not beneficial for property owners. Or maybe you argue that eminent domain is useful in certain circumstances, but not in others.

When you decide whether you want your synthesis essay to defend, challenge, or qualify that claim, you need to convey that stance clearly in your thesis statement. You want to avoid simply restating the claim provided in the prompt, summarizing the issue without making a coherent claim, or writing a thesis that doesn't respond to the prompt.

Here's an example of a thesis statement that received full points on the eminent domain synthesis essay:

Although eminent domain can be misused to benefit private interests at the expense of citizens, it is a vital tool of any government that intends to have any influence on the land it governs beyond that of written law.

This thesis statement received full points because it states a defensible position and establishes a line of reasoning on the issue of eminent domain. It states the author's position (that some parts of eminent domain are good, but others are bad), then goes on to explain why the author thinks that (it's good because it allows the government to do its job, but it's bad because the government can misuse its power.)

Because this example thesis statement states a defensible position and establishes a line of reasoning, it can be elaborated upon in the body of the essay through sub-claims, supporting evidence, and commentary. And a solid argument is key to getting a six on your synthesis essay for AP Lang!

Looking for help studying for your AP exam? Our one-on-one online AP tutoring services can help you prepare for your AP exams. Get matched with a top tutor who got a high score on the exam you're studying for!

Step 4: Create a Bare-Bones Essay Outline

Once you've got your thesis statement drafted, you have the foundation you need to develop a bare bones outline for your synthesis essay. Developing an outline might seem like it's a waste of your precious time, but if you develop your outline well, it will actually save you time when you start writing your essay.

With that in mind, we recommend spending 5 to 10 minutes outlining your synthesis essay . If you use a bare-bones outline like the one below, labeling each piece of content that you need to include in your essay draft, you should be able to develop out the most important pieces of the synthesis before you even draft the actual essay.

To help you see how this can work on test day, we've created a sample outline for you. You can even memorize this outline to help you out on test day! In the outline below, you'll find places to fill in a thesis statement, body paragraph topic sentences, evidence from the sources provided, and commentary :

  • Present the context surrounding the essay topic in a couple of sentences (this is a good place to use what you learned about the major opinions or controversies about the topic from reading your sources).
  • Write a straightforward, clear, and concise thesis statement that presents your stance on the topic
  • Topic sentence presenting first supporting point or claim
  • Evidence #1
  • Commentary on Evidence #1
  • Evidence #2 (if needed)
  • Commentary on Evidence #2 (if needed)
  • Topic sentence presenting second supporting point or claim
  • Topic sentence presenting three supporting point or claim
  • Sums up the main line of reasoning that you developed and defended throughout the essay
  • Reiterates the thesis statement

Taking the time to develop these crucial pieces of the synthesis in a bare-bones outline will give you a map for your final essay. Once you have a map, writing the essay will be much easier.

Step 5: Draft Your Essay Response

The great thing about taking a few minutes to develop an outline is that you can develop it out into your essay draft. After you take about 5 to 10 minutes to outline your synthesis essay, you can use the remaining 30 to 35 minutes to draft your essay and review it.

Since you'll outline your essay before you start drafting, writing the essay should be pretty straightforward. You'll already know how many paragraphs you're going to write, what the topic of each paragraph will be, and what quotations, paraphrases, or summaries you're going to include in each paragraph from the sources provided. You'll just have to fill in one of the most important parts of your synthesis—your commentary.

Commentaries are your explanation of why your evidence supports the argument you've outlined in your thesis. Your commentary is where you actually make your argument, which is why it's such a critical part of your synthesis essay.

When thinking about what to say in your commentary, remember one thing the AP Lang synthesis essay prompt specifies: don't just summarize the sources. Instead, as you provide commentary on the evidence you incorporate, you need to explain how that evidence supports or undermines your thesis statement . You should include commentary that offers a thoughtful or novel perspective on the evidence from your sources to develop your argument.

One very important thing to remember as you draft out your essay is to cite your sources. The AP Lang exam synthesis essay prompt indicates that you can use generic labels for the sources provided (e.g. "Source 1," "Source 2," "Source 3," etc.). The exam prompt will indicate which label corresponds with which source, so you'll need to make sure you pay attention and cite sources accurately. You can cite your sources in the sentence where you introduce a quote, summary, or paraphrase, or you can use a parenthetical citation. Citing your sources affects your score on the synthesis essay, so remembering to do this is important.

body-green-arrow-down

Keep reading for a real-life example of a great AP synthesis essay response!

Real-Life AP Synthesis Essay Example and Analysis

If you're still wondering how to write a synthesis essay, examples of real essays from past AP Lang exams can make things clearer. These real-life student AP synthesis essay responses can be great for helping you understand how to write a synthesis essay that will knock the graders' socks off .

While there are multiple essay examples online, we've chosen one to take a closer look at. We're going to give you a brief analysis of one of these example student synthesis essays from the 2019 AP Lang Exam below!

Example Synthesis Essay AP Lang Response

To get started, let's look at the official prompt for the 2019 synthesis essay:

In response to our society's increasing demand for energy, large-scale wind power has drawn attention from governments and consumers as a potential alternative to traditional materials that fuel our power grids, such as coal, oil, natural gas, water, or even newer sources such as nuclear or solar power. Yet the establishment of large-scale, commercial-grade wind farms is often the subject of controversy for a variety of reasons.

Carefully read the six sources, found on the AP English Language and Composition 2019 Exam (Question 1), including the introductory information for each source. Write an essay that synthesizes material from at least three of the sources and develops your position on the most important factors that an individual or agency should consider when deciding whether to establish a wind farm.

Source A (photo) Source B (Layton) Source C (Seltenrich) Source D (Brown) Source E (Rule) Source F (Molla)

In your response you should do the following:

  • Respond to the prompt with a thesis presents a defensible position.
  • Select and use evidence from at least 3 of the provided sources to support your line of reasoning. Indicate clearly the sources used through direct quotation, paraphrase, or summary. Sources may be cited as Source A, Source B, etc., or by using the description in parentheses.
  • Explain how the evidence supports your line of reasoning.
  • Use appropriate grammar and punctuation in communicating your argument.

Now that you know exactly what the prompt asked students to do on the 2019 AP Lang synthesis essay, here's an AP Lang synthesis essay example, written by a real student on the AP Lang exam in 2019:

[1] The situation has been known for years, and still very little is being done: alternative power is the only way to reliably power the changing world. The draw of power coming from industry and private life is overwhelming current sources of non-renewable power, and with dwindling supplies of fossil fuels, it is merely a matter of time before coal and gas fuel plants are no longer in operation. So one viable alternative is wind power. But as with all things, there are pros and cons. The main factors for power companies to consider when building wind farms are environmental boon, aesthetic, and economic factors.

[2] The environmental benefits of using wind power are well-known and proven. Wind power is, as qualified by Source B, undeniably clean and renewable. From their production requiring very little in the way of dangerous materials to their lack of fuel, besides that which occurs naturally, wind power is by far one of the least environmentally impactful sources of power available. In addition, wind power by way of gearbox and advanced blade materials, has the highest percentage of energy retention. According to Source F, wind power retains 1,164% of the energy put into the system – meaning that it increases the energy converted from fuel (wind) to electricity 10 times! No other method of electricity production is even half that efficient. The efficiency and clean nature of wind power are important to consider, especially because they contribute back to power companies economically.

[3] Economically, wind power is both a boon and a bone to electric companies and other users. For consumers, wind power is very cheap, leading to lower bills than from any other source. Consumers also get an indirect reimbursement by way of taxes (Source D). In one Texan town, McCamey, tax revenue increased 30% from a wind farm being erected in the town. This helps to finance improvements to the town. But, there is no doubt that wind power is also hurting the power companies. Although, as renewable power goes, wind is incredibly cheap, it is still significantly more expensive than fossil fuels. So, while it is helping to cut down on emissions, it costs electric companies more than traditional fossil fuel plants. While the general economic trend is positive, there are some setbacks which must be overcome before wind power can take over as truly more effective than fossil fuels.

[4] Aesthetics may be the greatest setback for power companies. Although there may be significant economic and environmental benefit to wind power, people will always fight to preserve pure, unspoiled land. Unfortunately, not much can be done to improve the visual aesthetics of the turbines. White paint is the most common choice because it "[is] associated with cleanliness." (Source E). But, this can make it stand out like a sore thumb, and make the gargantuan machines seem more out of place. The site can also not be altered because it affects generating capacity. Sound is almost worse of a concern because it interrupts personal productivity by interrupting people's sleep patterns. One thing for power companies to consider is working with turbine manufacturing to make the machines less aesthetically impactful, so as to garner greater public support.

[5] As with most things, wind power has no easy answer. It is the responsibility of the companies building them to weigh the benefits and the consequences. But, by balancing economics, efficiency, and aesthetics, power companies can create a solution which balances human impact with environmental preservation.

And that's an entire AP Lang synthesis essay example, written in response to a real AP Lang exam prompt! It's important to remember AP Lang exam synthesis essay prompts are always similarly structured and worded, and students often respond in around the same number of paragraphs as what you see in the example essay response above.

Next, let's analyze this example essay and talk about what it does effectively, where it could be improved upon, and what score past exam scorers awarded it.

To get started on an analysis of the sample synthesis essay, let's look at the scoring commentary provided by the College Board:

  • For development of thesis, the essay received 1 out of 1 possible points
  • For evidence and commentary, the essay received 4 out of 4 possible points
  • For sophistication of thought, the essay received 0 out of 1 possible points.

This means that the final score for this example essay was a 5 out of 6 possible points . Let's look more closely at the content of the example essay to figure out why it received this score breakdown.

Thesis Development

The thesis statement is one of the three main categories that is taken into consideration when you're awarded points on this portion of the exam. This sample essay received 1 out of 1 total points.

Now, here's why: the thesis statement clearly and concisely conveys a position on the topic presented in the prompt--alternative energy and wind power--and defines the most important factors that power companies should consider when deciding whether to establish a wind farm.

Evidence and Commentary

The second key category taken into consideration when synthesis exams are evaluated is incorporation of evidence and commentary. This sample received 4 out of 4 possible points for this portion of the synthesis essay. At bare minimum, this sample essay meets the requirement mentioned in the prompt that the writer incorporate evidence from at least three of the sources provided.

On top of that, the writer does a good job of connecting the incorporated evidence back to the claim made in the thesis statement through effective commentary. The commentary in this sample essay is effective because it goes beyond just summarizing what the provided sources say. Instead, it explains and analyzes the evidence presented in the selected sources and connects them back to supporting points the writer makes in each body paragraph.

Finally, the writer of the essay also received points for evidence and commentary because the writer developed and supported a consistent line of reasoning throughout the essay . This line of reasoning is summed up in the fourth paragraph in the following sentence: "One thing for power companies to consider is working with turbine manufacturing to make the machines less aesthetically impactful, so as to garner greater public support."

Because the writer did a good job consistently developing their argument and incorporating evidence, they received full marks in this category. So far, so good!

Sophistication of Thought

Now, we know that this essay received a score of 5 out of 6 total points, and the place where the writer lost a point was on the basis of sophistication of thought, for which the writer received 0 out of 1 points. That's because this sample essay makes several generalizations and vague claims where it could have instead made specific claims that support a more balanced argument.

For example, in the following sentence from the 5th paragraph of the sample essay, the writer misses the opportunity to state specific possibilities that power companies should consider for wind energy . Instead, the writer is ambiguous and non-committal, saying, "As with most things, wind power has no easy answer. It is the responsibility of the companies building them to weigh the benefits and consequences."

If the writer of this essay was interested in trying to get that 6th point on the synthesis essay response, they could consider making more specific claims. For instance, they could state the specific benefits and consequences power companies should consider when deciding whether to establish a wind farm. These could include things like environmental impacts, economic impacts, or even population density!

Despite losing one point in the last category, this example synthesis essay is a strong one. It's well-developed, thoughtfully written, and advances an argument on the exam topic using evidence and support throughout.

body-number-four-post-it-note

4 Tips for How to Write a Synthesis Essay

AP Lang is a timed exam, so you have to pick and choose what you want to focus on in the limited time you're given to write the synthesis essay. Keep reading to get our expert advice on what you should focus on during your exam.

Tip 1: Read the Prompt First

It may sound obvious, but when you're pressed for time, it's easy to get flustered. Just remember: when it comes time to write the synthesis essay, read the prompt first !

Why is it so important to read the prompt before you read the sources? Because when you're aware of what kind of question you're trying to answer, you'll be able to read the sources more strategically. The prompt will help give you a sense of what claims, points, facts, or opinions to be looking for as you read the sources.

Reading the sources without having read the prompt first is kind of like trying to drive while wearing a blindfold: you can probably do it, but it's likely not going to end well!

Tip 2: Make Notes While You Read

During the 15-minute reading period at the beginning of the synthesis essay, you'll be reading through the sources as quickly as you can. After all, you're probably anxious to start writing!

While it's definitely important to make good use of your time, it's also important to read closely enough that you understand your sources. Careful reading will allow you to identify parts of the sources that will help you support your thesis statement in your essay, too.

As you read the sources, consider marking helpful passages with a star or check mark in the margins of the exam so you know which parts of the text to quickly re-read as you form your synthesis essay. You might also consider summing up the key points or position of each source in a sentence or a few words when you finish reading each source during the reading period. Doing so will help you know where each source stands on the topic given and help you pick the three (or more!) that will bolster your synthesis argument.

Tip 3: Start With the Thesis Statement

If you don't start your synthesis essay with a strong thesis statement, it's going to be tough to write an effective synthesis essay. As soon as you finish reading and annotating the provided sources, the thing you want to do next is write a strong thesis statement.

According to the CollegeBoard grading guidelines for the AP Lang synthesis essay, a strong thesis statement will respond to the prompt— not restate or rephrase the prompt. A good thesis will take a clear, defensible position on the topic presented in the prompt and the sources.

In other words, to write a solid thesis statement to guide the rest of your synthesis essay, you need to think about your position on the topic at hand and then make a claim about the topic based on your position. This position will either be defending, challenging, or qualifying the claim made in the essay's prompt.

The defensible position that you establish in your thesis statement will guide your argument in the rest of the essay, so it's important to do this first. Once you have a strong thesis statement, you can begin outlining your essay.

Tip 4: Focus on Your Commentary

Writing thoughtful, original commentary that explains your argument and your sources is important. In fact, doing this well will earn you four points (out of a total of six)!

AP Lang provides six to seven sources for you on the exam, and you'll be expected to incorporate quotations, paraphrases, or summaries from at least three of those sources into your synthesis essay and interpret that evidence for the reader.

While incorporating evidence is very important, in order to get the extra point for "sophistication of thought" on the synthesis essay, it's important to spend more time thinking about your commentary on the evidence you choose to incorporate. The commentary is your chance to show original thinking, strong rhetorical skills, and clearly explain how the evidence you've included supports the stance you laid out in your thesis statement.

To earn the 6th possible point on the synthesis essay, make sure your commentary demonstrates a nuanced understanding of the source material, explains this nuanced understanding, and places the evidence incorporated from the sources in conversation with each other. To do this, make sure you're avoiding vague language. Be specific when you can, and always tie your commentary back to your thesis!

body-person-arrows-next

What's Next?

There's a lot more to the AP Language exam than just the synthesis essay. Be sure to check out our expert guide to the entire exam , then learn more about the tricky multiple choice section .

Is the AP Lang exam hard...or is it easy? See how it stacks up to other AP tests on our list of the hardest AP exams .

Did you know there are technically two English AP exams? You can learn more about the second English AP test, the AP Literature exam, in this article . And if you're confused about whether you should take the AP Lang or AP Lit test , we can help you make that decision, too.

Want to improve your SAT score by 160 points or your ACT score by 4 points?   We've written a guide for each test about the top 5 strategies you must be using to have a shot at improving your score. Download them for free now:

Trending Now

How to Get Into Harvard and the Ivy League

How to Get a Perfect 4.0 GPA

How to Write an Amazing College Essay

What Exactly Are Colleges Looking For?

ACT vs. SAT: Which Test Should You Take?

When should you take the SAT or ACT?

Get Your Free

PrepScholar

Find Your Target SAT Score

Free Complete Official SAT Practice Tests

How to Get a Perfect SAT Score, by an Expert Full Scorer

Score 800 on SAT Math

Score 800 on SAT Reading and Writing

How to Improve Your Low SAT Score

Score 600 on SAT Math

Score 600 on SAT Reading and Writing

Find Your Target ACT Score

Complete Official Free ACT Practice Tests

How to Get a Perfect ACT Score, by a 36 Full Scorer

Get a 36 on ACT English

Get a 36 on ACT Math

Get a 36 on ACT Reading

Get a 36 on ACT Science

How to Improve Your Low ACT Score

Get a 24 on ACT English

Get a 24 on ACT Math

Get a 24 on ACT Reading

Get a 24 on ACT Science

Stay Informed

Get the latest articles and test prep tips!

Follow us on Facebook (icon)

Ashley Sufflé Robinson has a Ph.D. in 19th Century English Literature. As a content writer for PrepScholar, Ashley is passionate about giving college-bound students the in-depth information they need to get into the school of their dreams.

Ask a Question Below

Have any questions about this article or other topics? Ask below and we'll reply!

All Subjects

2024 AP English Language and Composition Exam Guide

11 min read • june 18, 2024

A Q

Your guide to the 2025 AP English Language and Composition exam

We know that studying for your AP exams can be stressful, but Fiveable has your back! We created a study plan to help you crush your AP English Language and Composition exam. This guide will continue to update with information about the 2025 exams, as well as helpful resources to help you do your best on test day.  Unlock Cram Mode  for access to our cram events—students who have successfully passed their AP exams will answer your questions and guide your last-minute studying LIVE! And don't miss out on unlimited access to our database of thousands of practice questions.

Format of the 2025 AP English Language and Composition exam

This year, all AP exams will cover all units and essay types. The 2025 AP English Language and Composition exam format will be:

  • 45 questions in 1 hour
  • 1 synthesis essay
  • 1 rhetorical analysis essay
  • 1 argument essay

Scoring Rubric for the 2025 AP Lang Essays

Synthesis Essay

  • 1 point for a defensible thesis that responds to the prompt
  • Max of 4 points for providing evidence from at least 3 sources that support the line of reasoning AND commentary that explains and analyzes the evidence
  • Sophistication : - 1 point any of the following: - Creating a nuanced argument - Showing the limitations of the argument - Making effective rhetorical choices - Employing a style that is vivid and persuasive Rhetorical Analysis Essay
  • 1 point for a defensible thesis that analyzes rhetorical choices
  • Max of 4 points for providing specific evidence AND consistently explaining how the evidence relates to the line of reasoning AND showing how the rhetorical choices contribute to the author's message .
  • Explaining the significance of the rhetorical choices ( rhetorical situation )
  • Explaining the complexities of the passage and their purpose
  • Employing a style that is vivid and persuasive Argument Essay
  • 1 point for a defensible thesis
  • Max of 4 points for providing specific evidence AND consistently explaining the relevance of that evidence.
  • Crafting a nuanced argument by identifying complexities
  • Explaining the limitations of the argument by placing it in a broader context
  • Making rhetorical choices to improve the argument
  • Employing a style that is vivid and persuasive Check out our study plan below to find resources and tools to prepare for your AP English Language and Composition exam.

When is the 2025 AP English Language and Composition Exam and How Do I Take It?

The exam will take place on Wednesday, May 14, 2025 at 8:00 AM your local time.  

How Should I Prepare for the AP Lang Exam?

  • First, take stock of your progress in the course so far. What areas have you excelled and which sections need more focus? Download the  AP English Language Cheatsheet PDF  - a single sheet that covers everything you need to know at a high level. Take note of your strengths and weaknesses!
  • Build your study plan to review every unit and question type, but focus most on the areas that need the most improvement and practice. We’ve put together this plan to help you study between now and May. This will cover all of the units and essay types to prepare you for your exam- - Practice essays are your best friends! The more essays you write, the more automatic the process will come, and the easier the AP exam will be!- - Try some of the past exam questions  [object Object]
  • We've put together the study plan found below to help you study between now and May. This will cover all of the units and essay types to prepare you for your exam. Pay special attention to the units that you need the most improvement in.
  • Study, practice, and review for test day with other students during our live cram sessions via  Cram Mode . Cram live streams will teach, review, and practice important topics from AP courses, college admission tests, and college admission topics. These streams are hosted by experienced students who know what you need to succeed.

Pre-Work: Set Up Your Study Environment

Before you begin studying, take some time to get organized.

🖥 Create a study space.

Make sure you have a designated place at home to study. Somewhere you can keep all of your materials, where you can focus on learning, and where you are comfortable. Spend some time prepping the space with everything you need and you can even let others in the family know that this is your study space. 

📚 Organize your study materials.

Get your notebook, textbook, prep books, or whatever other physical materials you have. Also, create a space for you to keep track of review. Start a new section in your notebook to take notes or start a Google Doc to keep track of your notes. Get yourself set up!

📅 Plan designated times for studying.

The hardest part about studying from home is sticking to a routine. Decide on one hour every day that you can dedicate to studying. This can be any time of the day, whatever works best for you. Set a timer on your phone for that time and really try to stick to it. The routine will help you stay on track.

🏆 Decide on an accountability plan.

How will you hold yourself accountable to this study plan? You may or may not have a teacher or rules set up to help you stay on track, so you need to set some for yourself. First, set your goal. This could be studying for x number of hours or getting through a unit. Then, create a reward for yourself. If you reach your goal, then x. This will help stay focused!

2025 AP Lang Study Guide

🚧 unit 1 foundations of rhetoric: analysis of the rhetorical situation and claims., big takeaways:.

Unit 1 is an introductory unit that lays the foundations for the reading skills associated with how to understand and analyze complex texts. Skills here include identifying the ASPECTS of a text, analyzing the claim given and the evidence used to support that claim, and determining the function of the “chunks” in the argument. Because the content in this unit is very foundational, it is looped throughout the rest of the course instruction.

Definitely do this:

📚 Read these study guides:

Unit 1 Overview: Claims, Reasoning, and Evidence

1.1 Identifying the purpose and intended audience of a text

1.2 Examining how evidence supports a claim

1.3 Developing paragraphs as part of an effective argument 🎥 Watch these videos:  

College Board’s Instructional Video:  Overview of The Rhetorical Situation .- Fiveable’s  How to Read Like an AP Student .- Rhetorical Analysis Thesis Statements - Rhetorical Analysis Body Paragraphs ✍️ Practice:

Use the  Fiveable ASPECTS Guidesheet to help you break down a complex text. 🗺 Can you identify these rhetorical devices?

You won’t be asked to name drop on the exam, but it can be helpful to use devices when discussing strategies. Try  this Quizlet to help prepare.

Unit 2 Foundations of Argument: Analysis of an author’s choices in appeals and evidence

Unit 2 is an introductory unit that builds onto the foundations of rhetorical ASPECTS and moves toward planning and writing your own arguments. This unit focuses on the relationships between subject, speaker, and message, including examination of the structure and purpose of the given argument. The unit then moves into the developing thesis statements and building your own arguments with a clear line of reasoning.

Unit 2 Overview: Organizing Information for a Specific Audience

2.1 Analyzing audience and its relationship to the purpose of an argument

2.2 Building an argument with relevant and strategic evidence

2.3 Developing thesis statements

2.4 Developing structure and integrating evidence to reflect a line of reasoning 🎥 Watch these videos:  

College Board’s Instructional Video:  Identify Rhetorical Situation in a Pre 20th Century Text . 

Fiveable’s video on  How to Find Rhetorical Devices 📰 Check out these articles:

Here’s a  list of recommended rhetorical devices with definitions and examples! ✍️ Practice:

Use the  Fiveable Rhetorical Precis Guidesheet to help you break down a complex text. 🗺 Can you identify these elements of practical argument?

You won’t be asked to name drop of the exam, but it can be helpful to use devices when discussing strategies. Try  this Quizlet to help prepare.

👥 Unit 3 Confluence: Synthesis of multiple sources in argumentation

Unit 3 approaches multiple perspectives in argument through the lens of synthesis (that’s FRQ 1). In this study, you learn to identify effective and faulty reasoning while integrating a variety of evidence from credible resources that is properly cited in an original text.

Unit 3 Overview: Perspectives and How Arguments Relate

3.1 Interpreting character description and perspective

3.2 Identifying and avoiding flawed lines of reasoning

3.3 Introducing and integrating sources and evidence

3.4 Using sufficient evidence for an argument

3.5 Attributing and citing references

3.6 Developing parts of a text with cause-effect and narrative methods 🎥 Watch these videos:

Fiveable’s Introduction into  Synthesis Essays and  How to Begin Your Argument

College Board’s Instructional Video:  Complexity in Argument . 🗺 Can you identify these elements of synthesis?

👀 Unit 4 Reasoning: Analysis of argument from introduction to conclusion

Unit 4 includes a greater depth of focus on the writing of effective arguments -- the line of reasoning created in the introduction, built with modes of discourse, and strengthened in the conclusion. An important note about these skills of argumentation is that they build toward all parts of every FRQ. 

Unit 4 Overview: How writers develop arguments, intros, and conclusion

4.1 Developing and connecting thesis statements and lines of reasoning

4.2 Developing introductions and conclusions

4.3 Adjusting an argument to address new evidence 🎥 Watch these videos:  

College Board’s Instructional Video:  Understanding a Line of Reasoning .

Fiveable’s  Effective Annotations . ✍️ Practice:  

Try Fiveable’s  Guide to LOR Body Paragraphs . 🗺 Can you identify the rhetorical modes?

You won’t be asked to name drop them on the exam, but it can be helpful to use devices when discussing strategies. Try  this Quizlet to help prepare.

🧐 Unit 5 Commentary and Analysis: Analysis of complex argument and intentional rhetoric

In Unit 5, the skills look at the minutiae involved in argumentation: development of the line of reasoning that produces strong commentary and maintains the primary claim through all parts of the writing. To achieve these goals, this unit includes a focus on transitions , modifiers , and qualifications for argumentative perspective .  

Unit 5 Overview

5.1 Maintaining ideas throughout an argument

5.2 Developing commentary throughout paragraphs

5.3 Using modifiers to qualify an argument and convey perspective

5.4 Using transitions 🎥 Watch these videos:  

Fiveable’s video on  How to Improve Analysis Part 1 and  Part 2 - As well as how to  Embed Quotes into Body Paragraphs - Rhetorical Analysis Body Paragraphs - Synthesis Essay Body Paragraphs - Argument Essay Body Paragraphs 📰 Check out these articles:

Tara Seale’s adaptation for  Creating a Line of Reasoning . ✍️ Practice:

Try Fiveable’s  Guide to LOR Body Paragraphs .

🏃‍♂️ Unit 6 Rhetorical Risks: Analysis of multiple perspectives, bias, and shifts with new evidence

In Unit 6, you will notice a direct link building on the ideas of Unit 3 as this instruction looks at position and perspectives while synthesizing information strategically to support a claim.  For greater depth, this unit moves to modify a current argument to include new evidence.

Unit 6 Overview: Position, Perspective, and Bias

6.1 Incorporating multiple perspectives strategically into an argument

6.2 Recognizing and accounting for bias

6.3 Adjusting an argument to new evidence

6.4 Analyzing tone and shifts in tone 🎥 Watch these videos:  

College Board’s Instructional Video:  Creating a Nuanced Argument . 

Fiveable’s video on  Tracking an Author’s Argument

🚀 Unit 7 Complex Argumentation: Analysis of effective arguments, including concession and refutation

The skills of Unit 7 are about putting all units of study together to look at the complexity of a given argument and the effectiveness of the pieces built into that argument.  Though many teachers will have addressed counterarguments, concessions, and refutations before reaching this unit, those skills are highly scrutinized in this segment of learning.

Unit 7 Overview: Successful and Unsuccessful Arguments

7.1 Examining complexities in issues

7.2 Considering how words, phrases, and clauses can modify and limit an argument

7.3 Examining how counterargument or alternative perspectives affect an argument

7.4 Exploring how sentence development affects an argument 🎥 Watch these videos:  

Fiveable’s video on  Arguments and Counterarguments

College Board’s Instructional Video:  How Argument Demonstrates Understanding . ✍️ Practice:

Check your progress with  Fiveable’s AP Language Skills Matrix .

📝 Unit 8 Style: Analysis of how style influences the audience movement

Unit 8 covers how to understand the influence style has on the audience , and the purpose behind each decision. By analyzing these various tactics, students are able to understand the author’s audience, and how to effectively persuade them. Style is an important part in connecting the rest of the course and understanding how the rhetorical choices and devices are used to accomplish a purpose.

Unit 8 Overview: Stylistic Choices

8.1 Choosing comparisons based on an audience

8.2 Considering how sentence development and word choice affect how the writer is perceived by an audience

8.3 Considering how all choices made in an argument affect the audience

8.4 Considering how style affects an argument 🎥 Watch these videos:  

Fiveable’s  Analysis of the Mindset of the Audience - College Board’s Instructional video:  Analyzing and Understanding the Audience 📰 Check out these articles:

College Board’s explanation of  Elements and Context for Style ✍️ Practice:

Review this quizlet on  Elements of Style for more practice.

✏️ Unit 9 Craft: Creation of your own complex argument with synthesis and rhetoric

The final unit of AP Language and Composition covers how to effectively form your own arguments by acknowledging and understanding complexities to create a nuanced and sophisticated argument. It focuses on your ability to comprehend and connect multiple sources to create a well reasoned, and detailed argument as well as how to add in your own rhetorical devices and choices to make your writing more persuasive and effective.   

Unit 9 Overview: Developing a Complex Argument

9.1 Strategically conceding, rebutting, or refuting information

9.2 Crafting an argument through stylistic choices like word choice and description 🎥 Watch these videos:  

Fiveable’s video on  Creating your own Synthesis Arguments

College Board’s video on  Complexities within Arguments and  How to Create a Nuanced Argument

Key Terms to Review ( 38 )

© 2024 fiveable inc. all rights reserved., ap® and sat® are trademarks registered by the college board, which is not affiliated with, and does not endorse this website..

What are your chances of acceptance?

Calculate for all schools, your chance of acceptance.

Duke University

Your chancing factors

Extracurriculars.

ap lang essays time

How to Write the AP Lang Argument Essay + Examples

What’s covered:, what is the ap language argument essay, tips for writing the ap language argument essay, ap english language argument essay examples, how will ap scores impact my college chances.

In 2023, over 550,148 students across the U.S. took the AP English Language and Composition Exam, and 65.2% scored higher than a 3. The AP English Language Exam tests your ability to analyze a piece of writing, synthesize information, write a rhetorical essay, and create a cohesive argument. In this post, we’ll be discussing the best way to approach the argumentative essay section of the test, and we’ll give you tips and tricks so you can write a great essay.

The AP English Language Exam as of 2023 is structured as follows:

Section 1: 45 multiple choice questions to be completed in an hour. This portion counts for 45% of your score. This section requires students to analyze a piece of literature. The questions ask about its content and/or what could be edited within the passage.

Section 2: Three free response questions to be completed in the remaining two hours and 15 minutes. This section counts for 55% of your score. These essay questions include the synthesis essay, the rhetorical essay, and the argumentative essay.

  • Synthesis essay: Read 6-7 sources and create an argument using at least three of the sources.
  • Rhetorical analysis essay: Describe how a piece of writing evokes meaning and symbolism.
  • Argumentative essay: Pick a side of a debate and create an argument based on evidence. In this essay, you should develop a logical argument in support of or against the given statement and provide ample evidence that supports your conclusion. Typically, a five paragraph format is great for this type of writing. This essay is scored holistically from 1 to 9 points.

Do you want more information on the structure of the full exam? Take a look at our in-depth overview of the AP Language and Composition Exam .

Although the AP Language Argument may seem daunting at first, once you understand how the essay should be structured, it will be a lot easier to create cohesive arguments.

Below are some tips to help you as you write the essay.

1. Organize your essay before writing

Instead of jumping right into your essay, plan out what you will say beforehand. It’s easiest to make a list of your arguments and write out what facts or evidence you will use to support each argument. In your outline, you can determine the best order for your arguments, especially if they build on each other or are chronological. Having a well-organized essay is crucial for success.

2. Pick one side of the argument, but acknowledge the other side

When you write the essay, it’s best if you pick one side of the debate and stick with it for the entire essay. All your evidence should be in support of that one side. However, in your introductory paragraph, as you introduce the debate, be sure to mention any merit the arguments of the other side has. This can make the essay a bit more nuanced and show that you did consider both sides before determining which one was better. Often, acknowledging another viewpoint then refuting it can make your essay stronger.

3. Provide evidence to support your claims

The AP readers will be looking for examples and evidence to support your argument. This doesn’t mean that you need to memorize a bunch of random facts before the exam. This just means that you should be able to provide concrete examples in support of your argument.

For example, if the essay topic is about whether the role of the media in society has been detrimental or not, and you argue that it has been, you may talk about the phenomenon of “fake news” during the 2016 presidential election.

AP readers are not looking for perfect examples, but they are looking to see if you can provide enough evidence to back your claim and make it easily understood.

4. Create a strong thesis statement

The thesis statement will set up your entire essay, so it’s important that it is focused and specific, and that it allows for the reader to understand your body paragraphs. Make sure your thesis statement is the very last sentence of your introductory paragraph. In this sentence, list out the key points you will be making in the essay in the same order that you will be writing them. Each new point you mention in your thesis should start a paragraph in your essay.

Below is a prompt and sample student essay from the May 2019 exam . We’ll look at what the student did well in their writing and where they could improve.

Prompt: “The term “overrated” is often used to diminish concepts, places, roles, etc. that the speaker believes do not deserve the prestige they commonly enjoy; for example, many writers have argued that success is overrated, a character in a novel by Anthony Burgess famously describes Rome as a “vastly overrated city,” and Queen Rania of Jordan herself has asserted that “[b]eing queen is overrated.”

Select a concept, place, role, etc. to which you believe that the term “overrated” should be applied. Then, write a well-developed essay in which you explain your judgment. Use appropriate evidence from your reading, experience, or observations to support your argument.

Sample Student Essay #1:

[1] Competition is “overrated.” The notion of motivation between peers has evolved into a source of unnecessary stress and even lack of morals. Whether it be in an academic environment or in the industry, this new idea of competition is harmful to those competing and those around them.

[2] Back in elementary school, competition was rather friendly. It could have been who could do the most pushups or who could get the most imaginary points in a classroom for a prize. If you couldn’t do the most pushups or win that smelly sticker, you would go home and improve yourself – there would be no strong feelings towards anyone, you would just focus on making yourself a better version of yourself. Then as high school rolled around, suddenly applying for college doesn’t seem so far away –GPA seems to be that one stat that defines you – extracurriculars seem to shape you – test scores seem to categorize you. Sleepless nights, studying for the next day’s exam, seem to become more and more frequent. Floating duck syndrome seems to surround you (FDS is where a competitive student pretends to not work hard but is furiously studying beneath the surface just like how a duck furiously kicks to stay afloat). All of your competitors appear to hope you fail – but in the end what do you and your competitor’s gain? Getting one extra point on the test? Does that self-satisfaction compensate for the tremendous amounts of acquired stress? This new type of “competition” is overrated – it serves nothing except a never-ending source of anxiety and seeks to weaken friendships and solidarity as a whole in the school setting.

[3] A similar idea of “competition” can be applied to business. On the most fundamental level, competition serves to be a beneficial regulator of prices and business models for both the business themselves and consumers. However, as businesses grew increasingly greedy and desperate, companies resorted to immoral tactics that only hurt their reputations and consumers as a whole. Whether it be McDonald’s coupons that force you to buy more food or tech companies like Apple intentionally slowing down your iPhone after 3 years to force you to upgrade to the newest device, consumers suffer and in turn speak down upon these companies. Similar to the evolved form of competition in school, this overrated form causes pain for all parties and has since diverged from the encouraging nature that the principle of competition was “founded” on.

The AP score for this essay was a 4/6, meaning that it captured the main purpose of the essay but there were still substantial parts missing. In this essay, the writer did a good job organizing the sections and making sure that their writing was in order according to the thesis statement. The essay first discusses how competition is harmful in elementary school and then discusses this topic in the context of business. This follows the chronological order of somebody’s life and flows nicely.

The arguments in this essay are problematic, as they do not provide enough examples of how exactly competition is overrated. The essay discusses the context in which competition is overrated but does not go far enough in explaining how this connects to the prompt.

In the first example, school stress is used to explain how competition manifests. This is a good starting point, but it does not talk about why competition is overrated; it simply mentions that competition can be unhealthy. The last sentence of that paragraph is the main point of the argument and should be expanded to discuss how the anxiety of school is overrated later on in life. 

In the second example, the writer discusses how competition can lead to harmful business practices, but again, this doesn’t reflect the reason this would be overrated. Is competition really overrated because Apple and McDonald’s force you to buy new products? This example could’ve been taken one step farther. Instead of explaining why business structures—such as monopolies—harm competition, the author should discuss how those particular structures are overrated.

Additionally, the examples the writer used lack detail. A stronger essay would’ve provided more in-depth examples. This essay seemed to mention examples only in passing without using them to defend the argument.

It should also be noted that the structure of the essay is incomplete. The introduction only has a thesis statement and no additional context. Also, there is no conclusion paragraph that sums up the essay. These missing components result in a 4/6.

Now let’s go through the prompt for a sample essay from the May 2022 exam . The prompt is as follows:

Colin Powell, a four-star general and former United States Secretary of State, wrote in his 1995 autobiography: “[W]e do not have the luxury of collecting information indefinitely. At some point, before we can have every possible fact in hand, we have to decide. The key is not to make quick decisions, but to make timely decisions.”

Write an essay that argues your position on the extent to which Powell’s claim about making decisions is valid. 

In your response you should do the following:

  • Respond to the prompt with a thesis that presents a defensible position. 
  • Provide evidence to support your line of reasoning. 
  • Explain how the evidence supports your line of reasoning. 
  • Use appropriate grammar and punctuation in communicating your argument.

Sample Student Essay #2:

Colin Powell, who was a four star general and a former United States Secretary of State. He wrote an autobiography and had made a claim about making decisions. In my personal opinion, Powell’s claim is true to full extent and shows an extremely valuable piece of advice that we do not consider when we make decisions.

Powell stated, “before we can have every possible fact in hand we have to decide…. but to make it a timely decision” (1995). With this statement Powell is telling the audience of his autobiography that it does not necessarily matter how many facts you have, and how many things you know. Being able to have access to everything possible takes a great amount of time and we don’t always have all of the time in the world. A decision has to be made with what you know, waiting for something else to come in while trying to make a decision whether that other fact is good or bad you already have a good amount of things that you know. Everyone’s time is valuable, including yours. At the end of the day the decision will have to be made and that is why it should be made in a “timely” manner.

This response was graded for a score of 2/6. Let’s break down the score to smaller points that signify where the student fell short.

The thesis in this essay is clearly outlined at the end of the first paragraph. The student states their agreement with Powell’s claim and frames the rest of their essay around this stance. The success in scoring here lies in the clear communication of the thesis and the direction the argument will take. It’s important to make the thesis statement concise, specific, and arguable, which the student has successfully done.

While the student did attempt to provide evidence to support their thesis, it’s clear that their explanation lacks specific detail and substance. They referenced Powell’s statement, but did not delve into how this statement has proven true in specific instances, and did not provide examples that could bring the argument to life.

Commentary is an essential part of this section’s score. It means explaining the significance of the evidence and connecting it back to the thesis. Unfortunately, the student’s commentary here is too vague and does not effectively elaborate on how the evidence supports their argument.

To improve, the student could use more concrete examples to demonstrate their point and discuss how each piece of evidence supports their thesis. For instance, they could discuss specific moments in Powell’s career where making a timely decision was more valuable than waiting for all possible facts. This would help illustrate the argument in a more engaging, understandable way.

A high score in the “sophistication” category of the grading rubric is given for demonstrating a complex understanding of the rhetorical situation (purpose, audience, context, etc.), making effective rhetorical choices, or establishing a line of reasoning. Here, the student’s response lacks complexity and sophistication. They’ve simply agreed with Powell’s claim and made a few general statements without providing a deeper analysis or effectively considering the rhetorical situation.

To increase sophistication, the student could explore possible counterarguments or complexities within Powell’s claim. They could discuss potential drawbacks of making decisions without all possible facts, or examine situations where timely decisions might not yield the best results. By acknowledging and refuting these potential counterarguments, they could add more depth to their analysis and showcase their understanding of the complexities involved in decision-making.

The student could also analyze why Powell, given his background and experiences, might have come to such a conclusion, thus providing more context and showing an understanding of the rhetorical situation.

Remember, sophistication in argumentation isn’t about using fancy words or complicated sentences. It’s about showing that you understand the complexity of the issue at hand and that you’re able to make thoughtful, nuanced arguments. Sophistication shows that you can think critically about the topic and make connections that aren’t immediately obvious.

Now that you’ve looked at an example essay and some tips for the argumentative essay, you know how to better prepare for the AP English Language and Composition Exam.

While your AP scores don’t usually impact your admissions chances , colleges do care a lot about your course rigor. So, taking as many APs as you can will certainly boost your chances! AP scores can be a way for high-performing students to set themselves apart, particularly when applying to prestigious universities. Through the process of self-reporting scores , you can show your hard work and intelligence to admissions counselors.

That said, the main benefit of scoring high on AP exams comes once you land at your dream school, as high scores can allow you to “test out” of entry-level requirements, often called GE requirements or distribution requirements. This will save you time and money.

To understand how your course rigor stacks up, check out CollegeVine’s free chancing engine . This resource takes your course rigor, test scores, extracurriculars, and more, to determine your chances of getting into over 1600 colleges across the country!

Related CollegeVine Blog Posts

ap lang essays time

Are you seeking one-on-one college counseling and/or essay support? Limited spots are now available. Click here to learn more.

How to Write the AP Lang Synthesis Essay with Example

September 5, 2023

If you’re highly interested in learning more about writing analysis, then chances are you enrolled in AP Lang. Essentially, AP Lang is an advanced course for high schoolers that combines interest and knowledge in English with critical thinking. In the class, students learn how to analyze and synthesize a variety of texts to construct well-reasoned arguments. If you take AP Lang, then you can opt to take the AP test at the conclusion of the school year. On the exam, students write the AP Lang synthesis essay to demonstrate their learned abilities. In this article, we’ll look at what the AP Lang synthesis essay requires and show an example to provide better understanding of what to expect on the exam.

AP Lang Exam Basics

The AP Lang exam is separated into two sections. In the first section, students have one hour to answer a series of 45 multiple-choice questions. Here, about half of the questions are based on passages students read. The other half are focused on the best revision techniques. Essentially, the answers for the latter 20-22 questions are geared toward revising mock essays.

In this article, however, we’ll focus mainly on the second part of the exam: the AP Lang synthesis essay.

In this second section, students have two hours and 15 minutes to write three essays of their own design. The three open-ended questions in this section are intended to be free-response and allow for a variety of approaches. Each question is intended to allow up to 40 minutes to complete.

For the AP Lang synthesis essay, students are presented with a scenario of the College Board’s design. The scenario will provide its own thesis statement. Usually, scenarios relate to real-world problems like environmental concerns, media, or government policies.

For each scenario, students are provided with 6-7 outside sources. These sources could be in the form of an image, visual graph, or written paragraph. For written paragraphs, the sources are usually no more than 500 words.

Students are then expected to incorporate at least 3-4 of these outside sources into their essay response. The outside sources are intended to be used as supporting evidence for the student’s chosen stance or argument. Students are able to either agree with or disagree with the thesis presented in the original scenario.

AP Lang Exam – Scoring

In the second part of the AP Lang exam, students can earn a possible 6 points on each essay. 1 point is earned for the development of a thesis. Up to 4 points can be earned for evidence and commentary. The final 1 point is earned for sophistication of thought.

AP Lang Exam – Takeaways

Ultimately, the goal of the AP Lang synthesis essay is not whether the student is “right” or “wrong” in their argument. The key is that students are able to reasonably and clearly support their argument using the provided sources as evidence .

The College Board looks for your ability to identify relationships between texts , form a coherent argument , and interpret external sources .

Synthesis Essay AP Lang Examples

If you’re not sure how the questions will look on the AP Lang synthesis essay section, we’ll provide an example. After the example, we’ll break down the strengths and weaknesses of the response. That way, you’ll have a better idea of what the College Board is looking for.

Additionally, the College Board has released previous AP Lang synthesis essay examples you can review. They even have essay questions as recent as 2022 . For further support, a scoring commentary and comments from the Chief Reader are also available to view. Additionally, there are other examples you can view from earlier years .

Note: A good strategy to study for the synthesis essay AP Lang exam is to review your rhetorical devices and literary devices . Understanding how these devices function can be essential in constructing a cohesive essay.

Synthesis Essay AP Lang Examples – Sample Question

Below is a sample question from the AP Lang synthesis essay and a response to the prompt. This question was taken directly from a 2022 exam . However, the response to the question will be originally crafted for the purpose of this newsletter. As well, all supporting evidence will be originally created and does not correspond to any previous test.

The Question

Since the early 2000s, the United States government and a number of corporations have sponsored initiatives to improve education in the STEM disciplines: science, technology, engineering, and mathematics. The emphasis on STEM subjects in elementary, secondary, and higher education reflects concerns that United States students are less proficient in these areas than are students in other countries. Additionally, there is a belief that mastery in STEM fields is now essential in order to join a highly technical and specialized workforce. However, not everyone is convinced that a STEM-focused curriculum is necessary and/or effective.

In your response you should do the following:

  • Respond to the prompt with a thesis that presents a defensible
  • Select and use evidence from at least three of the provided sources to support your line of Indicate clearly the sources used through direct quotation, paraphrase, or summary. Sources may be cited as Source A, Source B, etc., or by using the description in parentheses.
  • Explain how the evidence supports your line of
  • Use appropriate grammar and punctuation in communicating your

How to Approach the Question

Maybe your first thought upon seeing this block of text is to feel overwhelmed. But don’t panic. There are effective ways to approach the question so you will be more prepared in your response.

It’s a good strategy to first isolate the thesis . What is the main idea of the text, and what is its argument?

Try it out. Reread the prompt and see if you can identify what the statement is asking you to develop an opinion on.

Think you’ve got it? In this example, we will be focusing on whether or not a STEM-focused curriculum in K-12 education is necessary and/or effective. In short, we will be arguing either for (highlighting the benefits) or against (highlighting the pitfalls) a STEM-focused curriculum.

How do we know what this statement is asking us?

Well, the statement provides a lot of background information. For example, we receive a definition of what STEM stands for. As well, we know that since 2000, there has been a greater initiative for STEM-focused classes.

When you read the prompt for the first time, it’s a great strategy to learn how to differentiate between background and contextual information from the heart of the argument .

A good way to learn how to isolate the argument is to look for transition words. Usually, these appear near the end of the question. Words like “however” and “yet” are signals that the statement is offering a differing opinion. Typically, the statement will tell you which two positions it’s offering for argument. These opinions are usually signaled by contrasting transition words.

So, now that we know what the question is asking us, what is the best way to respond?

Synthesis Essay AP Lang Examples – Sample Answer

The following is an essay response I crafted to the above question. After reading the sample, I will break down what it does well and what areas can be improved.

A STEM-focused curriculum is not as essential to providing a meaningful K-12 education. Because the majority of high school students are not proficient in STEM-focused classes, prioritizing these classes causes harm to student’s mental health and academic performance.

As seen in Source A, 60% of high school seniors in the Midwest only scored a C average in math and science-based classes (Langston). This statistic suggests that the majority of students do not resonate with STEM classes and therefore perform poorly. Earning a low score in any class does not bode well for students’ mental health.

When looking at the primary argument in Source C, it’s clear that most high schoolers prefer creative outlets to fact-based research (Kohler). Allowing students the opportunity to be more creative and initiate conversations about coursework lets students be more active in their learning. When students can discuss the nuance in their opinions, more personal growth happens. These conversations are not always easy to have in STEM-focused classes.

As well, when looking back to Source A, it’s clear that high school students in the Midwest earned higher grades, on average, in their English and art classes (Langston). This figure suggests that students perform better in these classes because they relate more to the source material. When relating to what they learn, they perform better in class.

In conclusion, STEM-focused curriculum is not as essential in K-12 education because most high school students do not relate to their STEM classes. When students do not earn satisfactory grades in these classes, it negatively affects their future college applications and job prospects.

Synthesis Essay AP Lang Examples – Answer Breakdown

So, what does this essay response get right, and where can it be improved? Let’s start with what the response does well.

First, the response establishes its thesis right away. Usually, it’s a good idea to clearly state your argument within the first paragraph. Not only is this a good practice because a reader can easily identify your stance, but also you can refer to your thesis as you write to make you stay on track.

With your thesis, it’s also a good idea to include one to two supporting sentences with the reasons why the thesis is concluded . Like in this example, I wrote that STEM-focused classes should not be prioritized because they can negatively affect both mental health and academic performance.

Another positive aspect of this response is that it is sure to not only reference but also cites its sources . It’s important that the reader understand where your information is coming from. That way, the readers can ensure you are interpreting the sources correctly.

AP Lang Synthesis Essay (Continued)

However, when rereading the instructions, it’s clear that this response fails the basic requirement of referring to at least three sources. Always make sure to reread the instructions to ensure you meet the standard requirements for incorporating source material.

Further, this AP Lang synthesis essay does not fully support its arguments . Ideas are simply stated and are not expanded upon.

For example, I mentioned a few times that earning low grades in STEM classes leads to negative mental health for high school students. However, there is no source referenced that either confirms or denies this claim. Therefore, there is no sufficient evidence to support my argument. It relies purely on inference.

Additionally, this AP Lang synthesis essay does not arrive at a sufficient level of sophistication of thought . Basically, sophistication of thought means avoiding broad generalizations and vague claims. The more specific you can be, the better your argument will sound.

Synthesis Essay AP Lang – In Conclusion

In the end, it’s always helpful to read the prompt thoroughly before writing. As well, making notes while you read could be a good strategy to pinpoint main ideas both in the prompt and the sources. That way, you can reread the material quickly. Similarly, sketching an outline may also be helpful. In addition, you should always carefully read the instructions to ensure all guidelines are followed.

As long as you avoid broad generalizations and use enough supporting evidence for your claim, you will be on the right path!

  • High School Success

Meghan Dairaghi

With a BA in English and an MFA in Creative Writing, Meghan has served as a writing tutor at the University of Missouri St. Louis and Maryville University. Additionally, Meghan has held editorial roles at River Styx and Boulevard, and was a prose reader at Farside Review . Most recently, her work has been featured in Belle Ombre , Flypaper Lit , and Mag 20/20 , among others, and she was nominated for the Mary Troy Prize in Fiction. 

  • 2-Year Colleges
  • ADHD/LD/Autism/Executive Functioning
  • Application Strategies
  • Best Colleges by Major
  • Best Colleges by State
  • Big Picture
  • Career & Personality Assessment
  • College Essay
  • College Search/Knowledge
  • College Success
  • Costs & Financial Aid
  • Data Visualizations
  • Dental School Admissions
  • Extracurricular Activities
  • Graduate School Admissions
  • High Schools
  • Homeschool Resources
  • Law School Admissions
  • Medical School Admissions
  • Navigating the Admissions Process
  • Online Learning
  • Outdoor Adventure
  • Private High School Spotlight
  • Research Programs
  • Summer Program Spotlight
  • Summer Programs
  • Teacher Tools
  • Test Prep Provider Spotlight

“Innovative and invaluable…use this book as your college lifeline.”

— Lynn O'Shaughnessy

Nationally Recognized College Expert

College Planning in Your Inbox

Join our information-packed monthly newsletter.

AP Research Topics: History, Chemistry, Psychology & More

ap lang essays time

Have you ever found yourself so wrapped up in a project that you forget to check the time? That's what happens when you choose the right AP Research topic. Whether you're curious about how technology shapes our lives or fascinated by the ways people think, picking good AP research paper topics can make your experience more than just another assignment—it can be an adventure.

AP Research is a course that lets high school students dig deep into a subject they care about. It gives you a chance to think critically and create something truly your own. But before you get started, you need to choose a topic that not only sparks your interest but also meets the course requirements. That's where we're here to help.

Need a Research Paper Upgrade?

Our experts will craft a paper that's smart, sharp, and totally stress-free.

What Does AP Research Do

AP Research allows you to take control of your learning. You'll learn how to ask the right questions, gather and analyze data, and present your findings in a way that's both clear and compelling.

One of the best parts is the freedom it offers. You're not just following a textbook—you're exploring something that genuinely interests you. This exploration can lead to impressive results, like the students who've used EssayPro's resources to write top-notch research papers. By connecting with experts and using reliable tools, they've turned their curiosity into polished projects that stand out.

Whether you're interested in social issues, science, or the arts, AP Research helps you develop skills that go beyond the classroom. You're able to solve problems and communicate your ideas effectively—all essential skills for college and beyond. And with the right support, like the guidance from EssayPro, you can take your research to the next level.

Don't miss out—check out our related article on why is critical thinking important for students and see how these skills can improve your AP Research process.

The List of AP Research Topics

Now that you know what this research is all about, it's time to find a topic that speaks to you. Having a good list of ideas can also make the process easier. Below, our expert admission essay services team gathered a selection of AP research paper topics that are not only interesting but also offer plenty of opportunities for deep exploration.

Best AP Research Topics

  • How do urban heat islands affect local weather patterns and public health?
  • The impact of early childhood education on long-term social and emotional development
  • What are the effects of climate change on global water resources and availability?
  • How do social movements utilize digital platforms to mobilize support and influence policy?
  • The role of genetic engineering in agriculture: benefits and ethical concerns
  • How do cultural differences influence the effectiveness of mental health interventions?
  • The effects of gamified learning on student engagement and retention in STEM subjects
  • How does the rise of remote work affect team dynamics and productivity?
  • The influence of historical narratives on modern national identities
  • What are the psychological and social impacts of long-term space travel on astronauts?
  • The role of artificial intelligence in personalized medicine and its potential for transforming healthcare
  • How do different teaching methods impact students with learning disabilities?
  • The impact of environmental sustainability practices on corporate social responsibility
  • What are the effects of food labeling on consumer behavior and dietary choices?
  • How do societal attitudes toward aging affect elderly care and policies?
  • The role of traditional knowledge in contemporary environmental conservation efforts
  • How do different approaches to urban design influence community well-being and social interaction?
  • The impact of digital detox practices on mental health and productivity in the workplace
  • What can be learned from studying the long-term effects of major historical pandemics on societies?
  • How do contemporary art movements challenge traditional notions of aesthetics and value?

Good AP Research Topics

Here are some more good topic choices curated by EssayPro. If you've found one you love, our experts are ready to offer help with writing an essay .

  • How does the design of public spaces impact community engagement and safety?
  • The influence of storytelling techniques on public awareness of social issues
  • What are the effects of dietary changes on cognitive function in older adults?
  • How do global supply chain disruptions affect local economies and businesses?
  • The impact of bilingual education on cognitive development and academic performance
  • How does exposure to different cultural perspectives shape attitudes toward global issues?
  • What are the psychological effects of participating in extreme sports or adventure activities?
  • How do social and economic factors influence access to clean drinking water in developing countries?
  • The role of artificial intelligence in detecting and preventing cyber threats
  • How do virtual communities influence real-world social interactions and relationships?
  • The impact of renewable energy adoption on rural versus urban areas
  • What are the effects of early intervention programs on children with developmental delays?
  • How do different types of exercise affect mental health and stress levels?
  • The influence of political satire on public opinion and political engagement
  • What can be learned from the success and failure of international environmental agreements?
  • How do personal finance education programs impact financial decision-making and stability?
  • The effects of digital media consumption on attention spans and learning abilities in adolescents
  • How does the portrayal of diversity in media affect societal attitudes and inclusion?
  • The role of community gardens in promoting local food security and social cohesion
  • What are the benefits and challenges of integrating technology into traditional classroom settings?

AP Seminar Topics

These AP Seminar research topics are sure to inspire and engage you. They're practical and interesting, perfect for exploring important issues.

  • How has the rise of influencer culture altered consumer trust in advertising?
  • The impact of urban farming initiatives on food security in densely populated cities
  • What role do emerging technologies play in the future of remote education?
  • How does the portrayal of mental health in media affect public perceptions and stigma?
  • The influence of digital privacy concerns on the development of new technologies
  • How do alternative justice systems, like restorative justice, impact community rehabilitation?
  • The effects of gamification on student motivation and learning outcomes in education
  • What can the evolution of language in internet memes tell us about cultural shifts?
  • How do local art movements contribute to global conversations on social justice?
  • The role of wearable technology in personal health monitoring and its implications for privacy
  • How does the concept of 'ethical fashion' challenge traditional fashion industry practices?
  • What are the psychological effects of participating in online communities versus physical communities?
  • The influence of microdosing psychedelics on creativity and productivity in professional settings
  • How do fictional narratives in popular media shape our understanding of historical events?
  • The impact of virtual reality therapy on treating phobias and PTSD
  • What role does citizen science play in advancing environmental research and policy?
  • How do different cultures approach the concept of work-life balance, and what can we learn from them?
  • The effects of digital art on the traditional art market and artist recognition
  • How does the rise of autonomous vehicles impact urban planning and traffic management?
  • The role of narrative in shaping public perceptions of scientific research and technology

AP World History Research Paper Topics

  • How did the Silk Road influence cultural exchange between Asia and Europe?
  • The impact of the Mongol Empire on global trade and communication
  • The role of women in ancient Egyptian society
  • What led to the fall of the Roman Empire?
  • Exploring the causes and effects of the Protestant Reformation
  • The significance of the Magna Carta in shaping modern democracy
  • How did the Black Death reshape European societies in the 14th century?
  • The rise and fall of the Byzantine Empire
  • What were the key factors in the success of the Ottoman Empire?
  • The influence of Confucianism on Chinese government and society
  • How did the Age of Exploration change global economies and cultures?
  • The impact of colonialism on Indigenous populations in the Americas
  • What were the causes and consequences of the French Revolution?
  • The development and spread of Islam during the Middle Ages
  • How did the Industrial Revolution transform societies in Europe and beyond?
  • The influence of Greek philosophy on Western thought
  • What were the driving forces behind the unification of Germany?
  • The role of the Transatlantic Slave Trade in shaping the modern world
  • How did World War I lead to major political changes in Europe?
  • The cultural and technological achievements of the Gupta Empire in India

AP US History Research Paper Topics

  • The causes and effects of the American Revolution
  • How did the Louisiana Purchase shape the future of the United States?
  • The role of women in the American Civil War
  • What were the main challenges faced by the early colonies in America?
  • The impact of the Emancipation Proclamation on the Civil War
  • How did the Gold Rush influence westward expansion in the United States?
  • The significance of the Monroe Doctrine in American foreign policy
  • What were the key factors leading to the Great Depression?
  • The influence of the Harlem Renaissance on American culture
  • How did the Civil Rights Movement change American society?
  • The causes and consequences of the Mexican-American War
  • What role did the New Deal play in America's recovery from the Great Depression?
  • The impact of World War II on American domestic life
  • How did the Cold War shape US foreign and domestic policies?
  • The significance of the Supreme Court case Brown v. Board of Education
  • What were the driving forces behind the Women's Suffrage Movement?
  • The effects of the Vietnam War on American society and politics
  • How did the Watergate scandal change public trust in government?
  • The influence of the Industrial Revolution on urbanization in America
  • What were the causes and outcomes of the American involvement in World War I?

AP Lang Research Paper Topics

  • How does the use of rhetorical questions in speeches influence audience engagement?
  • The role of persuasive language in shaping public opinion during elections
  • How does the choice of narrative perspective affect a reader's connection to a story?
  • The impact of social media on modern journalism and news reporting
  • What are the linguistic techniques used in effective political debates?
  • How do advertising slogans use language to create brand identity and consumer loyalty?
  • The influence of Shakespeare's use of imagery on modern literary analysis
  • How does the use of irony in literature enhance thematic elements?
  • The role of dialogue in character development in contemporary novels
  • How does language in public health campaigns affect community behavior and awareness?
  • What are the rhetorical strategies used in motivational speeches to inspire action?
  • The impact of tone and mood in setting the atmosphere in Gothic literature
  • How do different genres of writing, such as satire and tragedy, affect reader perception?
  • The role of metaphor in political rhetoric and its impact on policy discussions
  • How does the structure of an argumentative essay influence its persuasiveness?
  • The effects of language simplification in educational materials on student comprehension
  • How do authors use symbolism to convey deeper meanings in their works?
  • The influence of cultural context on the interpretation of literary texts
  • What are the effects of direct vs. indirect speech on character relationships in drama?
  • How do historical speeches reflect the values and concerns of their time periods?

Don't forget to buy analytical essay if you've nailed down your ideal topic!

AP Environmental Science Research Paper Topics

  • The impact of deforestation on global biodiversity
  • How does climate change affect polar ice caps and sea levels?
  • The role of renewable energy in reducing carbon emissions
  • What are the environmental consequences of plastic pollution in oceans?
  • The effects of industrial agriculture on soil health
  • How does urbanization contribute to habitat loss and species extinction?
  • The importance of wetlands in maintaining ecological balance
  • What are the challenges and benefits of sustainable farming practices?
  • The role of environmental policies in protecting endangered species
  • How does air pollution impact human health in urban areas?
  • The significance of the ozone layer in protecting life on Earth
  • What are the environmental impacts of fracking on water resources?
  • The effects of climate change on coral reef ecosystems
  • How does deforestation contribute to climate change?
  • The importance of conservation efforts in preserving biodiversity
  • What are the environmental benefits and challenges of electric vehicles?
  • The role of national parks in protecting natural resources
  • How do invasive species disrupt local ecosystems?
  • The impact of overfishing on marine life and ocean health
  • What are the environmental and social implications of e-waste?

AP Chemistry Research Paper Topics

  • The process of photosynthesis and its chemical significance
  • What are the effects of heavy metals on human health?
  • The chemistry behind pharmaceuticals and drug development
  • How do detergents and soaps work at the molecular level?
  • The significance of oxidation-reduction reactions in energy production
  • What are the chemical principles behind climate change mitigation efforts?
  • The role of catalysts in speeding up chemical reactions
  • How do acids and bases interact in everyday life?
  • The importance of the periodic table in modern chemistry
  • What are the environmental impacts of chemical fertilizers?
  • The chemistry behind renewable energy sources like solar cells
  • How do chemical bonds determine the properties of substances?
  • The significance of pH in maintaining biological systems
  • What are the applications of nanotechnology in medicine?
  • The process and importance of water purification techniques
  • How do greenhouse gases contribute to global warming?
  • The role of chemical reactions in food preservation
  • What makes enzymes so crucial in biochemical reactions?
  • The chemistry of batteries and how they store energy
  • How do polymers impact everyday products and the environment?

AP Biology Research Topics

  • How do genetic mutations contribute to evolution?
  • The role of enzymes in cellular processes
  • What are the effects of climate change on animal migration patterns?
  • The significance of the human microbiome in health and disease
  • How do plants adapt to extreme environmental conditions?
  • The impact of antibiotic resistance on public health
  • What are the mechanisms of gene expression and regulation?
  • The role of natural selection in shaping species diversity
  • How do hormones regulate growth and development in organisms?
  • The process of photosynthesis and its importance in the carbon cycle
  • What are the effects of habitat destruction on biodiversity?
  • The role of the immune system in defending against pathogens
  • How do organisms maintain homeostasis in varying environments?
  • The impact of invasive species on native ecosystems
  • What are the genetic factors involved in inherited diseases?
  • The process of cell division and its significance in growth and reproduction
  • How do environmental toxins affect the nervous system?
  • The role of symbiotic relationships in ecosystems
  • What are the effects of pollution on aquatic life?
  • The significance of epigenetics in gene expression and inheritance

AP Research Psychology Topics

  • How do different parenting styles impact child development?
  • The effects of sleep deprivation on cognitive performance
  • What are the psychological impacts of social media addiction?
  • The role of genetics vs. environment in determining intelligence
  • How does mindfulness meditation influence stress levels?
  • The effects of early childhood trauma on adult mental health
  • What are the cognitive benefits of bilingualism?
  • The impact of exercise on mental health and mood
  • How do stereotypes and prejudices develop and affect behavior?
  • The role of attachment theory in understanding romantic relationships
  • What are the psychological effects of prolonged isolation?
  • The influence of parental involvement on academic achievement
  • How do cognitive biases affect decision-making?
  • The effects of music therapy on anxiety and depression
  • What are the psychological mechanisms behind placebo effects?
  • How does exposure to violent media influence aggression?
  • The role of self-esteem in coping with life challenges
  • What are the effects of nutrition on cognitive function?
  • How does early intervention in autism spectrum disorder affect developmental outcomes?
  • The impact of stress management techniques on overall well-being

AP Capstone Research Topics

  • How did the rise of digital nomadism change the concept of work-life balance?
  • The impact of virtual reality on empathy and understanding of social issues
  • How do small, community-based conservation efforts contribute to global environmental change?
  • The role of crowdfunding in transforming startup culture and innovation
  • What can ancient agricultural practices teach us about modern sustainable farming?
  • How do storytelling techniques in video games influence player behavior and decision-making?
  • The effects of cross-cultural exchanges on traditional art forms in the digital age
  • What are the social and psychological impacts of living in a hyper-connected world?
  • How do grassroots movements influence national policy changes?
  • The role of biohacking in personal health and ethical considerations
  • How do urban green spaces impact community well-being and social interactions?
  • The influence of speculative fiction on real-world scientific advancements
  • What are the psychological effects of experiencing extreme weather events on communities?
  • How do traditional storytelling methods in indigenous cultures address modern social issues?
  • The role of augmented reality in enhancing educational experiences and learning outcomes
  • How do cultural festivals contribute to local economic development and global awareness?
  • The impact of immersive theatre on audience engagement and social change
  • What can historical patterns of migration reveal about current refugee crises?
  • How do alternative economic models, like time banking, challenge traditional concepts of value and work?
  • The effects of digital detox programs on mental health and productivity

AP English Language Research Paper Topics

  • How does rhetoric influence public opinion in political speeches?
  • The role of satire in social and political commentary
  • How do language and style vary between formal and informal writing?
  • The impact of social media on modern communication practices
  • How does persuasive writing shape consumer behavior in advertising?
  • The use of metaphor in shaping cultural narratives
  • What are the linguistic features of effective storytelling in literature?
  • How does the choice of diction affect the tone of a piece?
  • The role of rhetorical devices in Martin Luther King Jr.'s speeches
  • How do different genres of writing influence reader perception?
  • The impact of globalization on language and communication styles
  • How does the structure of a persuasive essay affect its effectiveness?
  • The use of imagery and symbolism in shaping the reader's emotions
  • How does the language used in news media influence public understanding of events?
  • The role of ethos, pathos, and logos in crafting compelling arguments
  • How does narrative voice affect reader engagement in memoirs and autobiographies?
  • The influence of historical context on the language of classical literature
  • How do language and rhetoric contribute to the development of national identity?
  • The effects of censorship on literary expression and freedom
  • How do authors use rhetorical strategies to address social justice issues?

Characteristics of a Good AP Research Paper

Strong AP research topics stand out due to several key features that ensure their impact. Here's what to keep in mind:

  • Timeless Relevance: Select a topic that maintains its significance over time. Avoid subjects likely to become outdated quickly. Instead, focus on issues with long-term importance that can be referenced by future researchers.
  • Supported by Credible Sources: Your research must be backed by reliable sources. For example, research supported by academic databases like JSTOR or publications from trusted institutions such as the National Institutes of Health (NIH) adds credibility.
  • Clear and Insightful Research Questions: Good questions help define the scope of your paper and shape your analysis, ensuring that your research is both relevant and insightful. For example, if your topic involves the impact of digital marketing, questions inspired by case studies from companies like Google or Adobe can provide a structured framework for your analysis.
  • Specific Focus: Narrow your topic to address specific issues, avoiding overly broad subjects. For instance, if researching the effects of climate change, focus on a particular aspect such as its impact on urban agriculture.
  • Logical Structure and Clarity: Ensure your paper has a clear introduction, body, and conclusion. Each section should flow logically, with well-organized arguments and evidence. A clear structure helps readers follow your argument and enhances the overall readability of your paper.

By considering these pointers suggested by EssayPro, your paper will make a valuable contribution to your field of study. It'll also be easy for you to draw on the best practices and examples from leading research institutions and industry experts.

Tips for Choosing an AP Research Topic

Here are some useful tips to help you choose standout AP research topic ideas:

  • Examine Current Trends: Look at recent news, technological advances, or social movements for inspiration. Topics like the effects of remote work on productivity or the rise of sustainable fashion can be both timely and impactful.
  • Reflect on Your Passions: Whether it's a hobby, a cause, or a field you're curious about, your enthusiasm will make the research process more enjoyable and your writing more compelling.
  • Seek Out Gaps in Existing Research: Identify areas where there's a lack of information or where current research is outdated. For example, if you notice few studies on the psychological effects of digital detoxes, that might be a unique angle worth exploring. EssayPro can also help you find existing research and identify gaps that need further investigation.
  • Consult with Experts: Talk to teachers, mentors, or professionals in fields you're interested in. Their insights can help you identify important topics. You might also find useful examples and advice from Essaypro's expert writers.
  • Consider Practical Applications: Choose a topic with real-world applications or implications. Researching the impact of new educational technologies on classroom dynamics, for example, can provide valuable insights and practical solutions.
  • Look at Your Local Community: Local issues or events can offer unique and relevant topics. Investigate how local policies affect community health or how a local environmental issue is being addressed. These topics can provide fresh perspectives and direct relevance.
  • Use Personal Experience: Leverage your own experiences or observations. If you've noticed a trend or issue in your daily life, such as the impact of social media on teen behavior, it can provide a unique and personal angle for your research.

Ready for a Paper That's More Brilliant Than Your Last Brainstorm?

Watch as our team transforms your rough ideas into a polished masterpiece that'll wow everyone.

How to Find a Good Research Topic in AP Research?

How long should my ap research paper be, what are some popular ap research topics.

Annie Lambert

Annie Lambert

specializes in creating authoritative content on marketing, business, and finance, with a versatile ability to handle any essay type and dissertations. With a Master’s degree in Business Administration and a passion for social issues, her writing not only educates but also inspires action. On EssayPro blog, Annie delivers detailed guides and thought-provoking discussions on pressing economic and social topics. When not writing, she’s a guest speaker at various business seminars.

ap lang essays time

is an expert in nursing and healthcare, with a strong background in history, law, and literature. Holding advanced degrees in nursing and public health, his analytical approach and comprehensive knowledge help students navigate complex topics. On EssayPro blog, Adam provides insightful articles on everything from historical analysis to the intricacies of healthcare policies. In his downtime, he enjoys historical documentaries and volunteering at local clinics.

Grey, S. (2024, August 7). What Is The AP Capstone Program? Everything You Should Know. Forbes . https://www.forbes.com/advisor/education/online-colleges/what-is-ap-capstone/

US History Topics

IMAGES

  1. AP Language Adversity Essay Free Essay Example

    ap lang essays time

  2. AP Lang

    ap lang essays time

  3. AP English Language and Composition Rhetorical Analysis Essay

    ap lang essays time

  4. AP Language and Composition argumentative essay

    ap lang essays time

  5. Ap Language And Composition Essay Samples

    ap lang essays time

  6. How to Get a 6 on Argument FRQ in AP® English Language

    ap lang essays time

VIDEO

  1. Why Was The Anarchist Cookbook Banned?!

  2. Stories of Secession

  3. Independent Reading for AP Lang and Comp

  4. Ap lang final project video essay

  5. Teaching Argument Writing

  6. AP Lang Q3 Evidence

COMMENTS

  1. AP English Language and Composition Exam

    AP English Language and Composition Exam - AP Central

  2. AP English Language and Composition Exam Questions

    AP English Language and Composition Exam Questions

  3. AP English Language and Composition Course

    Course Overview. AP English Language and Composition is an introductory college-level composition course. Students cultivate their understanding of writing and rhetorical arguments through reading, analyzing, and writing texts as they explore topics like rhetorical situation, claims and evidence, reasoning and organization, and style.

  4. Tackling the AP English Language and Composition essays: part 1

    #1 Organizing your time. On the AP Lang exam, you get a total of 2 hours and 15 minutes to write your three essays. This time is split into chunks. First, there is a 15 minute "reading period"; next, there is a 2 hour "writing period."

  5. AP Lang Exam FAQ

    How long is the AP Lang test? The first portion of the test is multiple choice. You have 1 hour to answer 45 questions. The second half is the essays. You have 2 hours and 15 minutes for 3 essays. This amounts to a 15-minute recommended reading period and 40 minutes for each of the essays.

  6. AP English Language and Composition

    Starting in the 2024-25 school year, AP English Language and Composition multiple-choice questions (MCQs) will have four answer choices instead of five. This change will take effect with the 2025 exam. All resources have been updated to reflect this change. Exam Duration.

  7. Ultimate Guide to the AP English Language and Composition Exam

    AP Language and Composition Score Distribution, Average Score, and Passing Rate. In 2019, 54.3% of the students who took the AP English Language and Composition exam received a score of 3 or higher. Only 9.9% of students who took the exam achieved the top score of 5, and 14.5% of students who took the exam scored a 1.

  8. Expert Guide to the AP Language and Composition Exam

    So each of the three free-response essays is worth about 18% of your score. As on other APs, your raw score will be converted to a scaled score of 1-5. This exam has a relatively low 5 rate. Only 10% of test takers received a 5 in 2022, although 56% of students received a score of 3 or higher.

  9. Guide to the AP English Language and Composition Exam

    Guide to the AP English Language and Composition Exam

  10. Expert Guide to the AP Language and Composition Exam

    1. Exam Format: The AP Language and Composition Exam consists of multiple-choice questions and free-response tasks. The multiple-choice section tests your reading comprehension and analysis skills, while the free-response section assesses your ability to write coherent and persuasive essays. 2.

  11. How to Write the AP Lang Rhetorical Analysis Essay (With Example)

    How to Write the AP Lang Rhetorical Analysis Essay (With ...

  12. How Long is the AP® English Language Exam?: Tips to Manage Your Time

    The AP® English Language exam has two sections. The total exam is three hours and 15 minutes long. The details of each section and part are outlined in the table below: Section I: Multiple Choice. 52 - 55 Questions. 1 hour. 45% of total exam score. Section II: Free-Response. 3 Questions.

  13. How to Write the AP Lang Argument Essay (With Example)

    Typically, the AP Lang Argument Essay prompt asks you to reflect on a broad cultural, moral, or social issue that is open to debate. For evidence, you won't be asked to memorize and cite statistics or facts. Rather, you'll want to bring in real-world examples of: Historical events. Current-day events from the news.

  14. Current/past AP Lang students have any advice on writing a good essay

    Leave out ~1 sentence from each body paragraph. This should give you time to get 3 sentences for your conclusion. If you finish in time then first of all congrats!!! Second, you can put an asterisk (*) in your body paragraph and take it on at the end, the AP exam graders are supposed to follow them and you can even do ** for a second one.

  15. How to Write a Perfect Synthesis Essay for the AP Language Exam

    How to Write a Perfect Synthesis Essay for the AP ...

  16. PDF AP® ENGLISH LANGUAGE AND COMPOSITION

    9 Essays earning a score of 9 meet the criteria for a score of 8 and, in addition, are especially sophisticated in their argument, thorough in development or impressive in their control of language. 8 Effective Essays earning a score of 8 effectively evaluate daylight saving time and offer a recommendation about its continued use.

  17. How to Write the AP Lang Synthesis Essay + Example

    Step 5: Write your Essay. Use the remaining 30-35 minutes to write your essay. This should be relatively easy if you took the time to mark up the sources and have a detailed outline. Remember to add special consideration and emphasis to the commentary sections of the supporting arguments outlined in your thesis.

  18. AP Lang Exam Guide

    Format of the 2025 AP English Language and Composition exam. This year, all AP exams will cover all units and essay types. The 2025 AP English Language and Composition exam format will be: Section I: Multiple Choice - 45% of your score 45 questions in 1 hour; Section II: Free Response Section - 55% of your score 2 hours and 15 minutes for:

  19. Behind the Scenes at the AP Exam (or: How to Grade 2,500 Essays in One

    This year was my fifth time scoring essays for the AP English Language and Composition exam, and it was nothing short of a fulfilling professional development experience. Approximately 530,000 students nationwide sat for this exam this year, which meant a total of 1.5 million essays. Roughly 1,500 readers had one week to score them all.

  20. How to Write the AP Lang Argument Essay + Examples

    How to Write the AP Lang Argument Essay + Examples

  21. How to Write the AP Lang Synthesis Essay with Example

    AP Lang Exam Basics. The AP Lang exam is separated into two sections. In the first section, students have one hour to answer a series of 45 multiple-choice questions. Here, about half of the questions are based on passages students read. The other half are focused on the best revision techniques.

  22. Synthesis Essay Materials

    Synthesis Essay Materials - AP Central - College Board

  23. AP Research Topics: EssayPro's Carefully Curated List

    Now that you know what this research is all about, it's time to find a topic that speaks to you. Having a good list of ideas can also make the process easier. Below, our expert admission essay services team gathered a selection of AP research paper topics that are not only interesting but also offer plenty of opportunities for deep exploration.

  24. PDF AP English Language and Composition

    AP ® English Language and Composition Sample Student Responses and Scoring Commentary Inside: Free-Response Question 1 ... Synthesis Essay 6 points . Since the early 2000s, the United States government and a number of corporations have sponsored initiatives to improve education in the STEM ... OR "Since the beginning of time . . ." ...